community member asks to hear from male employees instead of me, saying “that’s above my pay grade,” and more

It’s five answers to five questions. Here we go…

1. Community member cuts me off and asks to hear from male employees instead

I and other colleagues at my workplace (local government) have been meeting informally with members of the public about a project in my community that I am the project manager for. These are conversational meetings. One community member (he comes to all the meetings) has started verbally “managing” my responses to questions in the meetings and interrupting me when I speak. No one else in the room is receiving this treatment from him. I am not sure if my gender is playing a role here, but it seems that way because all of my colleagues in the room are male and older. There are other cues that are leading me to this conclusion, but for the sake of brevity I’ll leave them out.

Here is an example. He asks a general question about a project detail to the room and as I begin to answer he will interrupt me and say, “Hey Rachel, let’s let Fred answer that” or “Hey Rachel, I was hoping Paul could answer that.” My manager is in the meetings and thinks the behavior is odd, but it’s making me feel uncomfortable. Should I ignore this behavior in the meetings? Should I try to talk to this member of the public privately? Should I fight back in the meetings by saying something like, “Hey John, I’m sure my manager will correct me if I get off track”? I want to remain professional, but I do not want to be a doormat either.

Yeah, it’s probably your gender. I mean, maybe it’s something else, but just going by basic probability and how much this happens to women, it’s probably your gender.

Ideally your manager would be stepping in and saying, “Rachel is our expert in X so let’s have her answer this” or “Rachel is highly qualified on this topic so please let her speak” or so forth. Are you up for asking your manager to do that? Alternately, you can do something similar yourself — saying, “Please let me finish” and “I’m the project manager for this so I’d like to answer,” etc.

I’d also flag what’s happening to the rest of the group so they can have your back on it — because ideally when this guy asks Fred or Paul to address something that you’re better positioned to answer, they should say, “Rachel’s the best one to speak to that, so let’s hear her answer.” They might be oblivious to what’s happening, but if you spell it out for them and explain how you’d like them to handle it, good colleagues will do that.

Read an update to this letter.

2. How can I say “that’s above my pay grade?”

I am not an attorney, but I support attorneys and I have been doing it for so long, I operate at a fairly high level. Because of this, I often get asked to prepare responses to high level questions or to review/interpret documents, which others with my same job title would generally not be asked to do (and would not be capable of doing). If I wasn’t there, other attorneys would be asked to work through these requests.

Everything I do is under an attorney’s supervision, so there is no unauthorized practice of law issue, but these advanced requests bog me down and prevent me from being able to keep up with the work that I am supposed to be doing. How can I politely point out when I’ve been given a task that would normally be too advanced for someone at my title, and would be more appropriate to delegate to an attorney? If something is truly so complex I don’t even know where to begin, or I know I don’t have the background to perform a meaningful or helpful analysis, it’s easy for me to say that right away. However, for most tasks that are above my level/position/pay grade, I probably can work through the request eventually, but it takes significant time. I already have an extremely heavy workload, so it is a major strain. In the past, I’ve had colleagues who would say “that’s beyond my pay grade,” but it always sounded really abrasive to me.

I think you might be over-thinking it! You’re feeling like because you could do it, you need to offer more justification of why you shouldn’t. Instead, treat it more like you treat things you definitely shouldn’t be doing — meaning just be matter-of-fact and cheerfully explain who they should direct the work to instead. So: “That’s actually something a lawyer should do — check with Jane or Cecil on it.” Or, “We have the lawyers take requests like that — see if you can get it on Jane or Cecil’s calendar.”

3. “Thank you” responses sent via group email

I get a ton of emails every day, as I’m sure everyone does. My pet peeve is Steve will send out an email to many people asking for something. One of the people will respond to everyone with the information. So far, so good.

What upsets me is that Steve will then respond to everyone with “Thank you, Joe!” Why do I need to see this? In my opinion, this is just one more mail I have to open and delete.

This is indeed one of the great mysteries of our age. Is it performative gratitude from Joe? An attempt at closing the loop so no one else responds? As with what fate befell Amelia Earhart and the whereabouts of Jimmy Hoffa, no one knows.

I do think, though, that there’s a divide between people who are annoyed that replies like this add to their overall number of incoming emails and people who see no burden in taking one second to delete those messages and don’t really care. Steve may simply be in the second group. (I am too! I don’t get why people reply-all with those messages, but they take no time to delete.)

4. Applicant submitted confidential info as a writing sample

We are hiring for a junior lawyer position. As part of the hiring process, applicants submit a writing sample. One of our applicants has submitted an unredacted internal memo she apparently wrote as a summer law clerk (while still a student) at a different firm. The information is privileged work product, which would be bad enough, but it is a memo discussing an ongoing case where our office is opposing counsel. Normally, if something privileged is mistakenly sent out, we notify the other party that they appear to have given us a document by mistake, and assure them that we have not read it. But I am wondering whether I should contact the applicant first (or in addition) to give her a heads up and/or explain to her why this was such a serious error?

Oh no! Yes, please do. It’s important enough that she needs to know.

5. Laid off one week before one-year anniversary

Recently, I was laid off after my company went through a significant reorg that resulted in about 100 positions being eliminated (which was more than 10% of the workforce at the time). The layoffs were a complete surprise and my boss had no idea. She had to message her second line boss to find out how her team was impacted. Also, the day they informed us was the last day for those impacted.

The real kicker here is that I was laid off one week before my one-year service anniversary. The severance package was set up in a way where everyone got one month pay plus one week for every year you were with the company. Also, any company contributions to your 401K do not vest until the one-year mark. So between the extra week severance pay and the unvested funds in my 401K, the total is not an insignificant amount of money (more than $5,000) that either I’m technically not eligible for or I’ll lose.

My boss asked HR if they could get my tenure updated to one year but that was not accepted. Then she talked to her second-line boss and he thinks there is something else they may be able to do. At the moment, I’m waiting for their response. If the answer is no, there isn’t anything they can do, should I consult an employment lawyer? I’m a bit concerned that a lawyer may be costly and that I may not have much of a case anyway. Also, I’m nervous that the time is running out on signing the severance agreement (I don’t want to end up with nothing). Is there anything I could have done differently?

If you’re in the U.S., it’s possible that your employer violated the WARN Act, which requires most employers with 100 or more employees to provide 60 days notice of mass layoffs (defined as 50 or more at one time) pay the equivalent amount of time in severance. So if they aren’t giving you at least 60 days severance, they could be in violation of it (but there are some exceptions too).

Other that that, usually the only legal angle on getting more severance is if you have a potential claim of harassment or discrimination, in which case you can often negotiate more money in exchange for releasing them from any future legal claims (the type of severance agreement they want you to sign right now typically includes that release).

Even if you think you don’t have a claim like that, people are sometimes surprised by what a lawyer can dig up with the right questions. So yes, it’s worth talking to a lawyer to see what your options are. An initial consultation might be all you need, and you can often get that free or low-charge. Here’s a directory of employee-side employment lawyers by state.

It’s also okay to tell your employer you need more time before signing so you can have a lawyer review the agreement. Releases of claims can be declared invalid if the employee didn’t have enough time to seek legal counsel so they’ll probably agree unless you’re asking for a really excessive amount of time or unless it’s already been weeks.

{ 448 comments… read them below }

  1. Magenta Sky*

    “Ideally your manager would be stepping in and saying, “Rachel is our expert in X so let’s have her answer this” or “Rachel is highly qualified on this topic so please let her speak” or so forth. ”

    Yeah, if that’s not happening, the manager is a bigger problem than the community member.

    1. Wendy*

      This is one of those things where people not in marginalized groups have the opportunity to actually make a difference! A lot of being an ally is knowing when NOT to speak up, or when to redirect the spotlight to someone with more standing to answer questions. In this case, it’s up to LW1’s male coworkers to curb this behavior since it’s clear this community member isn’t going to listen to LW1 about it. It would be nice if they were able to recognize the issue for themselves, but sometimes they just aren’t looking for it and need a heads-up. Hopefully they’ll do something with the knowledge.

    2. Forrest*

      The only reason I can think of for the manager NOT doing this is if there’s a sense that the community members are supposed to be in control of the meeting, not the project team. But in that case, is there a community member chairing the meeting that you could speak to?

      If it’s really informal, there’s nobody chairing and you’re really trying to avoid any hierarchy, you need to speak to Paul and Fred and make sure they won’t take the floor when you’ve been shouted down, and then be really rigorous about calmly and assertively calling out the behaviour. (I would practise this in advance because it’s so hard.) Keep your voice very low and level and say, “Please let me finish, I am the best person to answer this.” Don’t let your voice get stressed, or higher, or angrier, because then other people will register it as “those two are shouting”. What you’re trying to do is trying to do is make it very clear to everyone there that HE is the one speaking over you.

    3. LadyAmalthea*

      When I had just started only female manager in an orthopedic shoe store where I had trained the 2 male managers in many of the more technical parts of the job, many times men, though rarely women, would not accept what I had said and would ask to speak to one of the male managers. It took me directly pointing out that I had told the customer the exact same thing word for word that they realized it was my gender and perceived age (looking young can be a serious professional disadvantage) that they knew to step in on a given signal and tell the customer I had trained them and knew what I was talking about better than anyone. Even if your colleagues mean well and are supportive, they often will need a very clear explanation as to what is going down when it comes to something they have never personally experienced by virtue of being male and not looking 23 at 37.

            1. Maxie's Mommy*

              Yep. Baby face and freckles. For years I wore non-prescription glasses to give my face some gravitas.

          1. many bells down*

            I’m almost entirely gray now and when I said it was my birthday a couple weeks ago, two of my co-workers guessed I was a good FIFTEEN YEARS younger than I am. These chubby baby cheeks of mine are never gonna age.

        1. Spencer Hastings*

          As a woman who’s prematurely greying and doesn’t look young for my age, I don’t think you have to be particularly young-looking to get ignored, dismissed, or talked to like you’re a child.

          1. SheLooksFamiliar*

            I’m a 60-something woman and it’s not as bad as it was when I was much younger, but yeah. I still get dismissed, talked over, and patronized.

          2. Rose*

            Of course you don’t have to be! You don’t have to be a women either. But neither helps. Do we really have to do the thing where we all point out “hey, people in my circumstance have also experienced hardship/discrimination?”

            1. M. from P.*

              I think what the commenters above are saying is that it’s more about gender than age.

            2. SheLooksFamiliar*

              We’re not ‘all’ pointing it out. Some of us are because, yes, we do feel the need to weigh in.

              In my case, it’s because the OP wrote, ‘…all of my colleagues in the room are male and older.’ I think it’s quite relevant to mention that this dynamic might not go away, or will only change slightly once she’s older.

              1. Sasha*

                This. I am a female physician in my 40s and look it, but have had male patients literally tell me to shut up, because they want to hear from my 18 year old but male medical student instead.

          3. Unaccountably*

            As someone who once did look young for my age, I can say with some authority that, no, looking young is not a requirement, but it certainly makes the problem worse. Belonging to multiple easily-dismissed groups does get you dismissed more often than people who belong to only one.

        2. Beatrice*

          I also have a baby face, and being not just female but a young-looking female DOES make a difference. What’s especially frustrating is when people tell me “oh, but you look so good!” or “oh, but you’ll be happy that you look young when you’re older!”

          Maybe so, but it’s a helluvan inconvenience NOW. Somehow, I doubt the ego boost from looking good in my 50s will balance out the frustrations from the 20s-40s. (I’m 40, and still have people thinking I’m fresh out of college.)

          1. JESUS IS THE MAN!*

            Yeah, if I were given the choice, I’d probably choose to be taken seriously as an adult now.

      1. straws*

        This – OP says the manager is in the meeting and finds it odd. I suspect the manager is male and picking up on something, but isn’t quite able to put his finger on it because he’s never experienced it before. There could be other issues with him, but if he’s already sensing something odd, a quick conversation to clarify might be enough for the light bulb to turn on and then he can take appropriate action.

        1. Hannah Lee*

          The other thing going on, that both the OP and the manager could strategize about, is that it seems one particular member of their community meetings is repeatedly pulling focus and derailing the meetings by trying to micromanage who from OP’s group is talking.

          Depending on who this guy is, what his role in the community is, it may be entirely appropriate to turn his own meeting moderation attempts back on him. “Howard, you’ve asked 3 questions in a row. Let’s give some other members of the community a chance to contribute to the discussion.” Because sometimes community meetings, condo board meetings, etc bring out that.one.guy. who loves the sound of his own voice or thinks he’s the king of the forest or has some other power wielding agenda and nothing is going to move forward until someone tells him to give it a rest and take his seat so someone else can have the mic for a change.

      2. Ace in the Hole*

        I work in a pretty niche department as part of a larger facility with many services. It’s a very male-dominated field… literally 99% men. My particular specialty involves a lot of safety and regulatory rules, and I’m one of only three people in the county qualified to do it. I trained the other two.

        Occasionally, a customer will decide they don’t want to listen to a young woman and will demand to speak with my manager. When that happens I call out my (big, gruff, bearded, male) manager. He’s a busy guy, so the customer has to wait a while. When Boss Dude finally shows up he introduces himself, listens to the customer, then turns to me and says, “Well Ace, you’re our expert in XYZ – what do you recommend?” The look on their faces when Boss Dude just repeats my instructions word for word is priceless.

        Boss Dude knows exactly what he’s doing, too.

    4. Brightwanderer*

      I think emphasising that LW1 is the project lead/has the expertise is a good line to take. My bet would bet that the way this works in this guy’s head is “ugh, LW1 always jumps in and answers before anyone else has a chance to talk” – which I suspect he would not be thinking if she were a man! Getting the manager to reinforce that LW1 is always the first to answer (if that’s even true – guys like that have a tendency to perceive any woman talking at all as “talking too much”…) because she’s the one leading the project strips away a big chunk of justification/plausible deniability.

      1. Pennyworth*

        Given the wording LW1 gives (“Hey Rachel, let’s let Fred answer that” or “Hey Rachel, I was hoping Paul could answer that”), it might be worth replying ”Would you mind telling me why?” and let them explain themselves.

        1. Squidlet*

          I find the wording* bizarre, in this context. It’s the type of thing you’d say as a facilitator when someone is hogging the spotlight and not giving other people a chance to speak.

          I don’t know the dynamics here but this guy comes over as aggressive and rude. It would be rude even if it wasn’t sexist, for the same person to be consistently “shushed”. And it’s pretty weird that OP’s manager and colleagues haven’t picked up on this, it doesn’t sound very subtle.

          * “I’d like Fred to answer this one”, etc

          1. generic_username*

            Agree. I almost wonder if the dude is or was a professional facilitator in some capacity. Regardless, it isn’t appropriate here

          2. Pants*

            Agree. Maybe it’s my age, but I’d have no issue calling that out at the moment. “Hey Mr. Dude. I’m sure you don’t realise, however but you frequently interrupt me and ask to hear from other men, when I am the SME on this. I can’t help but notice you don’t interrupt any of the men in this meeting for any reason. I’m bringing this to your attention for two reasons. One, I have valid and valuable knowledge on the subjects we are speaking on, which is why I’m in this meeting. Two, I would appreciate you not interrupting me going forward.”

            If you emphasise “MEN” enough, he’ll get the point. If not, call that out as well. “Mr. Dude, I’ve asked you not to interrupt me and you’ve done it again. This appears to me to be a gender bias. Would you like to explain why you have continued to interrupt me, the only woman in the room, but not the other men in this meeting?”

            As I get older, my middle fingers get higher in the air. Come at me, bro.

            1. Three Goblins in a Trench Coat*

              This is exactly how I would handle it — call it out calming and directly every time. No sugarcoating allowed. However, I’m also aware that I’m a mid-40s woman with some gray in my temples and a long, respected reputation in my field. I have the gravitas and f*ck-around-and-find-out confidence that I never would have had in my 20s and 30s. That’s why I’m the first to aggressively stick up for younger women in the workplace.

              1. Aunty Fox*

                I love being in my 40’s as I am finding this sort of confidence comes much easier now. :)

              2. laowai_gaijin*

                Being in your 40s is great. If I had the confidence of my 40s and the body of my 20s, I could rule the world.

                1. Three Goblins in a Trench Coat*

                  Me too! I wish I could impart that level of confidence on my younger self, but I’d probably be too powerful.

              3. Pants*

                I’m also in my 40s. There’s something about turning 40 that erodes the patience with f*ckery. I still look young, thanks to oily skin and purple hair, so it can come as a bit of a shock when the “finding out” part comes into play. (I love that phrase so much.) It’s irritating to have to stand up to this crap for myself and other women (I do that too), but it’s also a little delicious when I see the colour drain from someone’s face when they thought I’d just apologise and quietly slink away. Not today, dude. Not today.

                1. Three Goblins in a Trench Coat*

                  I love this! I had pink hair for most of 2020/21 and boy did it ever throw older men for a loop when they found out I hate teeth. I don’t look particularly young but the colored hair and nose ring confused their radar.

              4. allathian*

                I hear you! I’m lucky enough to work in an environment where this sort of misogynistic dismissal is rare, if not quite non-existent. But I do admit that it happens less and less now that I’m firmly middle-aged. My youthful smooth skin is balanced out by the natural gray in my hair, and my weight to some extent by the sort of self confidence that comes from liking myself enough that I don’t particularly care if others like me or not, so I won’t jump through hoops to be thought likable anymore, the way I did in my 20s and early 30s.

                I strive to be professional at all times at work, and being thought approachable is a part of that. But that doesn’t mean that I’d expect to have to put up with bad behavior from others to do my job well.

      2. Richard Hershberger*

        Team up with one the men in the group, “Bob.” Bob will answer the question by turning to LW, she will answer it, then Bob repeats her answer word for word. Repeat as necessary. The community member is ridiculous. Make it obvious that everyone realizes this.

        1. Napkin Thief*

          This is exactly what I had in mind! Make it not only obviously ridiculous but incredibly inconvenient for this guy to keep trying to pass you over.

        2. generic_username*

          I wouldn’t have Bob repeat her answer – I’d have him say “LW’s answer was complete so I have nothing to add” or something like that. It’s a fairly well-known phenomena that things get taken more seriously if a man says them, even if a woman literally just said it. This will just train the community member to expect Bob’s confirmation of LW’s correctness by repeating her response

          1. Dasein9*

            Except this trains everyone to see Bob as having the capacity and authority to approve or disapprove what LW says, which reinforces the sexism.

            1. Amaranth*

              Yes, that was my concern about LW’s idea of offering to have her manager step in if she needs correcting. It makes it sound like she is not actually the SME, but being indulged or trained, and might reinforce the reasoning that its better to go straight to the ‘real’ experts.

        3. The Cosmic Avenger*

          Or Bob could say something like “Well, actually, [this is Rachel’s area of expertise|Rachel oversaw that aspect of the project], so I would normally consult with her in order to answer that question. Rachel?” It is stupid and unnecessary, but if another male defers to her, the sexist boor might take the hint more readily.

    5. BritChickaaa*

      Despite Alison referring to the manager as “she”, LW explicitly stated that her manager is male.

      Being the only woman in a room when you’re dealing with hostile and sexist participants is really tough.

    6. Bagpuss*

      The other thing you can do is turn it back on him and treat it on the assumption that he’s simply forgotten / doesn’t know the project / you expertise or is looking for reassurance
      “Hey Rachel, let’s let Fred answer that”
      “Bob, perhaps you’ve forgotten, but I’m actually the project manager in this one, so I’m best placed to answer your question”

      But definitely speak to the others and ask them to support you – which could just be :
      Him “Hey Rachel, let’s let Fred answer that”
      Fred “No, Rachel’s got it, she’s very familiar with the project / she’s an expert on this project”

      1. Lexie*

        I wonder what Bob’s reaction would be if she asked him why when he says he wants to hear from someone else.

        1. Cee*

          Yeah the only innocent answer I can think of is that LW is steamrolling these meetings without realizing it and Bob is trying to give other people a chance to talk.

          But given the details here, its probably her gender. I do wonder whether Bob would admit to that or if he even is aware of his own motivation.

          1. Office Lobster DJ*

            Yeah, it’s probably LW’s gender. There are other possibilities — the one I came up with is that Bob wants Fred to trip himself up on something or Bob wants the assurance that the whole team and not just the PM knows what’s going on — but probably her gender.

          2. Unaccountably*

            Women are perceived as dominating conversations or meetings when they hit about 30% as much speech time as men. (I don’t remember the reference for this study off the top of my head.) So it could feasibly be both – LW could be talking much less than Fred is, but because of her gender, Bob perceives her as talking too much and not letting Fred do his fair (80%) share of the talking.

            1. Artemesia*

              the manager chairing the meeting should not be ceding management of the meeting to rude dude. He needs to say, ‘Jim, Rachel is our team lead on this and in the best position to answer that question. Rachel. . ‘

              But if this is actually an issue of Rachel dominating the meeting by answering everything, (and I realize that this may absolutely not be the case) then SHE could re-direct an occasional question.

              It really feels though like a case where this jerk is controlling a meeting that should be managed by the chair and so he is not only annoying and bossing the OP around but also usurping the manager’s role.

              1. CatPrance*

                If I’m the project lead and the subject matter expert, yes, I WILL be answering the majority of the questions, if not all of them. I am, after all, the project lead and the subject matter expert.

                That’s important.

          3. Amaranth*

            I think it would be valuable for LW to talk to her colleagues with an open mind, though, and mention it seems to be gender motivated, but its also possible something about her voice or speech makes it difficult for Bob to comprehend — if its an accent/speed/volume he might be embarrassed to admit he can’t follow along.

    7. kittymommy*

      Truthfully the biggest problem I see is that the manager/staff should not be letting a community member/citizen direct the course of the meeting. While a community meeting is done for the benefit of the citizens, it is organized and led by the project managers, in this case the local government. Staff needs to be the ones taking the reins of the meeting and the manager (or whomever is the highest ranking staff in the room) needs to get the room under control. If this has been a continual problem, and it sounds like it is, this group needs a stronger leader.

      1. Former Radio Guy*

        Exactly, and as a former member of the media who used to cover local government at the municipal level, I’ve seen numerous examples of “concerned citizens” who attempt to hijack public meetings and intimidate public officials. Well run governments with good elected officials can nip this in the bud, but badly run local governments can descend into utter chaos. I’ve seen both examples.

    8. Momma Bear*

      I agree that the men on the project need to start redirecting the questions back to OP if she’s actually the best one to answer. The community member is not being “odd”, he’s being a jerk. Women get this behavior all the time. IMO even if you want to tread carefully in the community, this is one guy. Think about everyone else watching (like other women) who might appreciate more of OP’s POV. Community Member sounds full of himself. Stop that behavior.

    9. JSPA*

      If this isn’t their first go-around with the squeaky wheel, they may have an established procedure, and want to keep OP out of his sights.

      I’ve been active enough at City Council Meetings that I notice which perennial gadflies are effectively assigned a council member or two who play “good cop” with them, while the others look at the ceiling or confer with aides (because when they interact directly, a stream of invective may be directed their way, which is…awkward…on the live stream). If he is indeed first name with two colleagues, this may be the dynamic here, as well.

      It’s also possible that history has shown the citizen which few of the public-facing figures can be counted on to say only what they know to be true, and follow through on what they say. I know I have a small number of people whom I already trust to give it to me non-evasive answers that check out. Any new person is an unknown; but when past experience has shown unknown to break 80% or more into the “not solidly dependable” category…then yes, I’d like to hear it from my regular trusted sources. (note: I am female, and the trusted sources do not skew disproportionately white nor male nor straight; this isn’t code for “old boy’s network.”)

      Finally, especially in planning, the new people often don’t know where the bodies are buried / how business is actually done. Because they are innocent of the subterfuge, they often don’t have the same “tells” (in the poker sense) as the long-timers.

      For all of these reasons, only and specifically in government–where you can’t get rid of your clients, but they can get rid of you, where everything is theoretically on record, but where the sausage still somehow keeps getting made behind the scenes–I would not automatically default to, “it’s anti-female bias, pure and simple.”

      1. DameB*

        I mean, yes, there are difficult “clients” in government and sometimes there are workarounds — I’m often the workaround in my office so I know exactly what you mean. But that fact don’t exclude anti-female bias — it’s just a way to work around some dude being an asshat and that asshatty-ness includes misogyny.

        And if they have a standard work around to deal with this Missing Stair, the other members of the team should say that to her. Not doing so is deeply problematic.

        1. JSPA*

          Sure, insofar as it’s policy and they’re aware of it, they should read the new people in, on some level. But

          a) they don’t necessarily want to say, explicitly, “we don’t really pay attention to That Guy, despite his charts and graphs, so ignore him and let Steve handle it, their kids went to school together.” (Being even slightly on record as ignoring a particular constituent is intrinsically problematic in its own right, and it’s functionally especially problematic when you don’t know how the new person will feel about that.)

          b) sometimes these work arounds are not explicit plans, but patterns that people have fallen into. There may be no explicit plan to share, and “reading OP in” may, in full, look like telling OP “don’t worry about it, That Guy’s just like that, Steve talks his language.”

          Point being, OP may be crediting That Guy with a lot more importance and pull than he’s got…or not. And how to proceed depends in part on getting a fuller sense of the lines of power. Otherwise, you can burn capital on making an ask that you afterwards wish you hadn’t made…especially if you end up as the designated Jerk Whisperer.

          1. CatPrance*

            It would not be a good idea to announce all that about Steve and the kids during the meeting when minutes are being taken, no, but there is a LOT of time in the week when they are not in that meeting. If there’s a lot of behind-the-scenes stuff and a lot of personal history between people that needs to be taken into account, they need to tell OP about that before the meetings.

            Has that happened? No, it hasn’t. So, while that may be a likely scenario that’s been outlined, in some situations, we’re still looking at a random male member of the community who’s trying to boss the female team lead/SME around and who cuts her off at every opportunity, during meetings, in favor of her male team members.

      2. The OTHER Other*

        This seems very much like discounting the LW’s experience. You say it may not be sexism, sometimes PITA people at community meetings are picky about who talks to them because others are inexperienced or untrustworthy. But there’s no indication here that LW is either of those things. Sometimes behavior that appears sexist really is just that, and there’s no need to invent explanations for why it might actually be something else.

        1. JSPA*

          You can set standards for a client; you can report a client’s representative to the head office; you can even drop a client if they keep being sexist.

          You can’t fire a constitutent or drop a constituent or tell them how to speak or whom to address, short of libel or threats. You can’t cut their allotted three minutes (or 10, if they signed up to speak to a proposal that’s before council, in advance) for telling you that you’re the illuminati and in cahoots with aliens, either. Or blaming you for their spouse leaving, or calling you a “blue assed baboon, I mean buffoon” or blaming you for the breakdown of the American Family. (Council meetings can be…wild.)

          In that context, someone demanding to talk to a specific other person or two is pretty low on the list of what people are expected to take, in fulfilling a governmental function.

          More generally, it’s really bimodal, whether being ignored in this way, and rolling with it, makes you look bad, or actually makes you look more professional–specifically due to this being a public facing governmental role.

          I’m not saying OP can’t or shouldn’t call it out. I’m not saying it’s not sexism. I am saying that OP may benefit from checking whether there are strategic decisions to be made here…and whether in this circumstance, doing one’s duty professionally sometimes very much includes making space for attitudinal or emotionally messy people to be attitudinal and emotionally messy in ways that keep them out of your hair.

          1. Hannah Lee*

            You may not be able to fire a client when you’re in government.

            But you, or whoever in your team is running the meeting, can, in fact, run the meeting. Which means moderating, structuring it in such a way that one citizen is not allowed to repeatedly derail the meeting by treating it as his own personal karaoke jukebox where HE and HE ALONE gets to select who speaks when. Unless he’s the ‘leader’ / spokesperson for the majority of your stakeholders, he has no more standing than any other citizen or the people presenting to direct the proceedings. And if this is an ongoing forum, it may need to be restructured as a whole, possible with each attendee able to ask one question and then they have to sit down, away from the stage any any microphones, to give someone else time and space to ask THEIR questions.

            Because this guy isn’t just undermining LW and derailing the meeting for her. He’s derailing the meeting and wasting the time of everyone in that room. The staff aren’t going to be able to accomplish what they need to in those sessions if this guy is allowed to keep doing this.

      3. Nameless in Customer Service*

        Why does it matter if the community member’s intent is or is not sexism?We’re not here to weigh his heart against the feather of truth, but to help LW #1 do her job in the face of his dismissiveness, not least because no matter what his reason his actions do map to and reinforce sexist patterns in society.

    10. Gov Gal*

      LW1 here. Love all the feedback and comradeship from my fellow fresh-faced women in their 40s. I had a talk with my manager and he agreed to try and help me with this community member in the next meeting. I have over a decade of experience in my area of expertise and second in command of our department. The work I do is very male dominated, so unfortunately I am familiar with this kind of behavior, but was a bit taken aback by a few callouts in front of and entire meeting room.

  2. Undine*

    #2 Closing the loop, and I actually appreciate it. It’s the difference between ‘Thanks Joe, that calendar has all the holiday info I need’, and ‘Thanks Joe, but I found that calendar doesn’t seem to have the festival of Moloch which I need to work around for our Carthegenian clients.’

    1. Dark Macadamia*

      LW3, would you rather potentially get multiple reply-alls confirming, adding to, or rewording the first answer? I’d be glad Steve is closing the loop promptly before more people decide to weigh in.

    2. Ness*

      Yep. The “thank you” email is a quick way of signifying, “my question is answered and no further information is needed.”

      1. Flying Wombat*

        Yep. If someone texted out a question to multiple people, I always appreciated the reply-all thank you. No thank you- my brain still has to keep that exchange in storage in case the asker didn’t see the answer, the asker didn’t get the answer they really wanted, or I missed part of the question that I was needed to answer. With a thank you- I can safely wipe the whole exchange out of my brain.

        1. Wendy*

          Additionally – if they aren’t known for closing the loop, they’ll be less likely to get answers to future questions because everyone will assume someone else already handled it.

          1. John Smith*

            In my workplace, this usually takes the form of a response with the subject line amended to include “now sorted out” or similar which means the email can be safely deleted/ignored without having to open it

          2. Lyudie*

            Being known for sending the “thank you!” loop-closing email helped me out once. A SME sent me some info, and when I didn’t reply, he IM’ed me to ask if I’d gotten his email. I had NOT gotten his email and it turned out there were some technical issues with the email that morning and many emails were lost. I’m sure we would have figured it out later, but his realizing that my silence was out of character was helpful in that case.

        2. just another bureaucrat*

          Yup. I work with a couple people who are very diligent about this and it’s nice I never worry about if their things have been fully closed or not.

          So yes, sometimes I’ll go back to a months old email and reply all asking if it’s been completed because if it hasn’t then it’s going to turn into something else and I need to manage that.

          It’s certainly not evil to close something fully.

    3. Allonge*

      Totally depends on the situation and corporate culture, but some loops are closed with the answer being sent. If Joe wanted file X, Amy sent file X. Joe is welcome to say thank you to Amy, but for everyone else, it’s not necessary.

      If it’s not a full answer, Joe needs to ask a specific question or, if I notice that something is missing, I need to respond even if Joe said thank you.

      1. Allonge*

        This assumes of course that Amy sent the answer as reply-all. If not, sure, close the loop.

          1. Allonge*

            This has to be a corporate-cultural divide: for me the expectation is that Amy replies-all, if I have any doubts I (or anyone else addressed) check it and add anything that needs to be added.

            If I see Amy replied and I don’t hear from Joe, I assume he got what he wanted. If he still has questions, it’s on him to say so.

            Fascinating, really. (Sorry, Star Trek weekend behind me).

            1. Rolly*

              Another colleague and I try to close loops with a message to the key person, and a BCC to everyone else (stated as “moving everyone else to BCC since Tammy and I have this”

      2. Mr. Shark*

        Yes, many times I’ll just reply to the direct sender with a “Thank you” rather than a reply-all. Sometimes I will reply-all because I want the direct sender’s manager (if they are on the e-mail) to know that the answer was sufficient and closed the loop.

    4. Kella*

      Yes this. Surprisingly often, people do not read the other replies to an original message and so won’t pay attention to the fact that the question was already answered. I find that if I send a “Thank you, Joe, for handling it” message or whatever, people are more likely to read an additional email from me and understand that the matter is solved.

      1. Seeking Second Childhood*

        Count me as glad for loop closers…even if too many people at my company read&reply in chronological order WITHOUT looking for replies & loop-closers.
        (This is a pet peeve of mine by now. These same people tend to answer with well thought out possible avenues of research when there was already a clear resolution. It was urgent 36 hours ago…give us the credit of thinking maybe we followed up already…just do a quick sort to see if there’s a reply saying it’s answered!)

        1. generic_username*

          Oh God, my boss is terrible about this when he comes back from vacation. He’ll come in on a Monday and resurrect something that was done (and not with anything new, to be clear – he’ll send the file that was requested and sent, or reply-all to say I’ll handle it when I already handled it three days before).

          1. OneAtATime*

            Oh yes! I have three peer level managers who do this. They work through their emails from oldest to newest and will not move to the next one until they have responded to the first. And they refuse to turn on “conversation” mode for their emails to group them together.

            So, we get something like:
            – Email out to everyone
            – Reply-all claiming responsibility and ending the reply-all thread (in theory)
            – Issue fully resolved, everyone has moved on.
            – Manager returns.
            – Replies all to the first email, “I don’t have information on this, can anyone else help requestor?”
            – Hour or so later, replies all to the claiming responsibility email (usually, “Thanks, Jane!”)
            – Hour of so later, replies all to their first reply all saying “Please disregard this email as Jane has it”)

            Or you know, you could maybe, gasp, read your emails and get caught up before taking action!

      2. Boof*

        It does confirm the request has been fulfilled – sometimes useful if you’re wondering whether to chime in!

        1. Office Lobster DJ*

          Sure, sometimes it’s not an e-mail you’ll be replying to right away, for whatever reason. In those cases, I’d rather spend a half second reading/deleting a “Thanks!” e-mail than have it take up space in my head as something to check back on and deal with later.

    5. Autumnheart*

      In addition to closing the loop, I honestly kinda feel like publicly acknowledging Joe for his action is a positive thing. Adding a little visibility to someone’s good work is a good thing to do (assuming the reply-alls don’t get out of hand). In a lot of work cultures, the default is to not thank anyone and to give praise only rarely, and that is kind of a bummer. And sometimes it leads to situations where management believes your performance is way lower than it actually is, because they don’t see all the stuff you actually DO in the course of a typical day. So I think having a little bit of a thank-you culture is a good thing.

      One alternative that you could try to encourage is to have the next Joe send the numbers just to you (not reply to everyone) and then you say “Joe sent me the numbers I needed. Thanks, Joe!” That at least saves some people from the email chain, while still closing the loop.

      1. Allonge*

        On your second paragraph: this for me is the worst of both worlds. The (presumably useful) information in the answer is going to one person, the thank you with low information value is going to all.

        I agree that we should be thanking people publicly; I disagree that sending a rote thank you for all actions is a good way to do this. If Joe’s answer is really useful, sure, point it out and thank him in front of all; if it’s something that happens five times a day, it’s too much for me.

        1. New Job So Much Better*

          Maybe it just lets everyone else know someone already supplied the answer, so that a hundred others don’t also send the info.

      2. generic_username*

        Depending on how active the chain is, I do a “Thanks Joe! And thank you to everyone else for your help on this as well!” It doesn’t quite work if Joe was the only person responding and everyone else was just CCed for visibility, but then presumably all those people knew Joe was the person who did the work and deserves the thanks

    6. Underrated Pear*

      Yes and I’ll add another view: my previous workplace had a very weird culture of no one responding to emails for, like, a week or so. You just sent a request into the ether and hoped it was eventually seen. My current organization is a place where people habitually reply-all in the way the LW describes. I love it because I know the conversation has been dealt with and is closed. Take it from me, it is FAR less time consuming to be in an environment where people take the not-entirely-necessary step of sending that signal. Yes, it means a few more emails to delete, but it also means people aren’t left wondering what’s been resolved on their team or not.

    7. anonymous73*

      It really depends on the situation, and an extra “thank you” isn’t always necessary.

    8. ThisIshRightHere*

      I got caught with this last week! I’d been part of an email chain that was up to maybe 20 replies. I saw from the thumbnail on my phone that someone replied “thanks…” (which I promptly ignored) and then the chain went dead. I thought we were done. Next workday I get reamed out for not chiming in on an update. I went back, actually opened up the “thanks” email and two lines down, somebody had shared some pertinent information that I totally missed. Smh.

    9. Unaccountably*

      There isn’t always a loop to be closed. If someone’s just sending out information (“IT is going to be doing repairs to the phone network Saturday from 4-6pm,” “The office supply order is now in the supply closet,”) surely an individual “Thank you” is fine and it doesn’t need to go to the entire list.

      1. Worldwalker*

        Does that even require an individual “thank you”? Those announcements are the email equivalent of tacking up a sign on the break room bulletin board. If one goes out to 100 people, and they each individually thank whoever sent it, the poor dude is going to have pages of “thank you” emails over a routine announcement.

    10. Tequila & Oxford Commas*

      Count me as another vote for closing the loop with a reply-all “thank you!” I’d rather take two seconds, literally, to read that and move on than to get another four emails from people all sending the same answer to the question that Joe already answered.

      1. SweetFancyPancakes*

        Exactly. If someone else has answered and their answer has been acknowledged, then I know it’s done and dusted and I don’t need to worry about it.

    11. WindmillArms*

      LW3, I have to admit I’m a compulsive “reply-all-thank-you” emailer! As discussed here, it’s to tell everyone who got the original message that the loop is closed. I also can’t always predict who is going to message me two days later asking if I got [answer], or what/where [answer]’s status is now. Sometimes there’s a higher-level person on the email thread who will “check back” out of the blue if no one replies-all to an email, and it seems to be pretty random when that happens.

      I do kind of agree that just the word “thanks” is a bit pointless. It has to be combined with either confirmation that the thing is resolved now (“Thanks! That’s exactly the file I needed.”) or that the answer is now available if the answerer only messaged me. (“Thanks for sending that! I posted the file on our SharePoint in case anyone needs it.”)

    12. Midwest Teacher*

      I use reply all to say thank you when someone didn’t reply all when answering my question. It signals to everyone that I got the response I needed and I don’t need anyone else to respond.

  3. Loulou*

    I feel absolutely sure that literally 100% of the people who have ever complained on or to AAM about having to delete emails spend significantly more time at work reading AAM than deleting emails! I get a ton of extraneous emails all day every day and it takes me genuinely under a second to decide to ignore each one…I just can’t wrap my head around caring about this. Free yourself, OP!

    1. Perfectly Particular*

      Most of our emails are subject to Legal Hold – so not allowed to be deleted. Filing all those thank yous, or leaving them in your inbox does take some mental energy. I’m sure there are others here in the same boat that can’t/won’t delete for whatever reason.

      1. ecnaseener*

        Most email clients let you set a keyboard shortcut to send an email to a specific folder. If you’ve been clicking and dragging everything, I can see how that’s annoying – set up a shortcut so it takes no longer than pressing delete.

        1. AnonInCanada*

          That’s why I do–I set up SHIFT-CTRL-1 to move items to a “Done with this so archive” folder and SHIFT-CTRL-2 to move to a “to do.” If it’s something not worth my time, the DELETE key works just fine as well :-). Just don’t forget to go into that trash folder and delete them all permanently once you know something didn’t get there by mistake.

      2. Julia*

        I’m really surprised anyone still ever deletes emails now that both Gmail and Outlook have an archive function and disk space limitations are no longer a problem. Are you guys really deleting emails, or just saying “delete” when you mean “archive”?

        1. quill*

          Oh no, random ads for adobe products that were purchased for my position and announcements that the sales manager for another country has retired get deleted right away. As do procedurally generated emails that say “llama pasture was unlocked for more than five minutes” when it’s llama poop shoveling day and that’s entirely expected.

        2. The OTHER Other*

          I think by “delete” they mean delete. I delete lots of emails I get, and I’m surprised archiving requirements people are talking about are relying on something as unreliable as the end user remembering what can’t be deleted, and what needs to be saved where. My industry requires archiving but it’s not something I have to do anything about, IT handles that on a server level. In that sense, nothing is ever really deleted, at least not until a required minimum number of years pass.

          1. womanaroundtown*

            Interesting… on outlook, it automatically archives for me if I am on my phone. So I always just swipe to archive any unwanted/unnecessary replies, and feel no frustration whatsoever.

      3. Accountant*

        Hitting the “archive” button takes mental energy that hitting the “delete” button does not? K

        1. The OTHER Other*

          I have never seen an “archive” button in email, either personal or for work, and anyway the comment seemed to suggest things needed to be saved in multiple different archive categories. I’m glad I’ve never had to do that.

          1. Worldwalker*

            I happen to have Gmail (work) open in another window and it’s above the email list, first one on the left.

            I also have Thunderbird up (personal) and it’s a right-click option. Which I need to use more.

    2. Workerbee*

      If we’re tossing out percentages, I would feel more assured that folks spending time reading AAM begin to see how there is no 100% rule, what with the myriad of nuances, cultures, industries, etc.

    3. TeamPottyMouth*

      For me, it’s not about the time it takes to delete the email (which is not zero, and is compounded multiple times over the workday), it’s about the number of interruptions. And yes, I have attempted to limit the number of times a day that I pause to read emails, but then I’m harassed by people who expected me to read their messages within seconds of delivery.

      1. AnonInCanada*

        I’m harassed by people who expected me to read their messages within seconds of delivery.

        Now that’s annoying!! Those people who think the world revolves around them and everyone should drop everything to cater to their needs. UGGGH! The more they push, the longer it takes me to respond. They’ll learn eventually :-P

    4. Lacey*

      I find it obnoxious because most of my emails need attention right away – so I have my email alert noise on.
      When I get a bunch of little pings over things that have nothing to do with my job, it is distracting and I do hate those people a little bit in the moment.

      Back when I had a job where email was something I caught up periodically as I had time, I could dismiss it with an eye-roll over how my coworkers didn’t know how to use email.

      Although a big difference is that none of these emails need a loop closed. More like, “Congratulations to Sandra for Achievement!” and the entire office replying all, “Congratulations Sandra!” all day long.

    5. Esmeralda*

      BEcause it’s over and over…. and over. And sometimes important info gets tucked into one of those reply-alls. I have to at least glance at the content. I have better things to do. I am being interrupted. And it adds up. A minute or two here, a minute or two there…along with time wasted dealing with spam and phishing that makes thru the spam filter, it’s enough to be a genuine PITA.

    6. Victoria*

      Do you know how long it took me to get my store management teams to use reply all? We were constantly getting a reply to just us, that included the info that the entire management team needed to know. We would then need to forward that info to the entire management team.

      This was a daily issue. I finally got them all to remember to use reply all, I am not gonna complain about the occasional email that I didn’t need to get.

    7. Elder Millennial*

      One they do for leisure and out of free will, one is forced upon them by others. That’s quite a different situation.

      That’s beside the point that others point out about being interrupted vs choosing out the moment to read something.

    8. Grey*

      Email to All: Announcement: Jane has been promoted to District Manager.

      100+ employees using Reply All throughout the day: Congratulations Jane!

      1. UseTheBCC!*

        This is why more people should use BCC! We are slowly training management to put the sender in the “to” field, anyone else who might need to see a reply in the “CC” field (so maybe “Jane” in the example), and everyone else in the company on the intracompany announcement in the “BCC” field.

        Email then reads:
        “BCC: Everyone in the office
        [because invariably, people will respond “I received this but was not on the email, why?”]

        Announcement: Jane has been promoted to District Manager.

        Congratulations, Jane!

        Sincerely,
        Sender”

      2. Cold and Tired*

        If you use Outlook, there’s an ignore function in the right click menu that I use in situations like this where I know I’m going to get spammed on an email chain I will never need to look at again. It just auto deletes all further emails in the conversation for me so I never have to see them. Obviously not great for anything that might be relevant for work, but great for the dumb mass emails that are just announcing things and get silly reply alls.

      3. Office Lobster DJ*

        Oh yes. This. And at a certain point enough people Reply All that you feel compelled to do it yourself, lest someone is noticing who performed Congratulations for Jane.

  4. MyDogIsCalledBradlyPooper*

    Thank You – Reply All

    I understand that this is just noise and clutters up your inbox is it worth being upset about or trying to change? For me, this fits in the “Things I Cannot Change” box so let it go. When I see these I just chuckle to myself and think “There goes Darren again making another email blunder”. Annoying = yes; Hurting Anyone = No; Worth Coaching Darren = Probably Not. He might have valid reasons for doing this. Or you look like your are trying to change his behaviour which in most cases is not part of your job.

    1. Allonge*

      For me it’s not so much being upset but wishing we could notify someone that X happened without six emails arriving in my mailbox (it’s here-thankyou-welcome-now on secondary location-thank you-welcome).

      If it’s a slow day, it’s a ‘shrug, Loreena cannot survive without being the last one sending in the exchange’. If it’s a busy day, I reeeeeaally don’t need the extra 4-5 emails. We all survive; a magic button to make it go away would be welcome.

      1. Elizabeth Bennett*

        YES. If email software would have an option to just delete any email that only says “thank you” in the message, that’d be a popular option!

        1. Worldwalker*

          Thunderbird filters could probably do it!

          I’m fairly sure that Thunderbird filters can do almost anything. I’m equally sure that I have filters dating back a decade to protect email accounts I no longer have from spammers that no longer exist. These two things are related. I need to clean up my filters.

    2. Smithy*

      In addition to the “not bothered” people – there are also a number of people who’ve had managers or workplaces that formally or informally encouraged this. So whether people just learn this from their formative working years and think it’s a must or aren’t bothered by it, I think it’s something that seeps in.

      I also find there are people who enjoy the performatively full inbox as part of a larger performative business piece. It’s often worse from people who genuinely are quite busy, but then will never hand over responsibilities either. So having a super full inbox (even if there’s a lot of reply all thank yous”), full calendar, etc. all feed something that they’re chasing.

      1. A Feast of Fools*

        This is my company, especially at the corporate level. We’re really big on showing appreciation for co-workers’ efforts, so if you reply with a Thank You only to the person who gave you the info, anyone else on the chain won’t see it and [in my experience] will start to think you’re selfish / self-centered / not a good company fit.

        So anytime I’m included in an email where a question is asked of someone else on the email, I’m already mentally prepared for the inevitable reply-all Thank You.

        1. The OTHER Other*

          At an old job in a call center, line managers would send an email to the whole department if someone received a complimentary call from a customer. It would always generate a flurry of “congratulations, Jane!” etc reply alls. But the worst thing was that there was no spellcheck on the subject line and inevitably the manager would misspell “complimentary”. It was interesting how MANY different ways it would be misspelled–“compelmentory”, “colpimentary” “coplementary” etc. All reproduced in the responses.

          I actually started a list to see how many different misspellings there’d be. The final tally was 14. Yes, it was petty. Could I let it go? No, I could not.

          1. Petty Speller*

            That’s hysterical, and making that list is totally something I’d do as well

            Reply-All is death by a thousand paper cuts

    3. Xantar*

      I actually think that in the situation described in #3, Steve’s “Thank you” email is not only appropriate but actually necessary and informative.

      Steve sent out an email to a bunch of people requesting information. Someone replied all with the requested information. Steve replied all, “Thank you!” In other words, Steve is saying, “I got your information, it answers my question, and everybody else on this email thread doesn’t need to spend any time writing an answer to me now.” This is good communication.

      Where things get annoying is if the original email is something the recipient isn’t necessarily interested in and there are a bunch of replies sent to everyone (“It’s Wanda’s five year anniversary at the company!” followed by “Congratulations Wanda” 37 times). Or if LW was included in the original email but never had any ability to answer it. But those are separate issues from the “thank you” email at the end.

    1. Wendy*

      If there was a way to retract the email after the question was answered, that would be better. Second-best would be a way to edit it saying “this is no longer something you need to worry about.” Failing that, though – something that is possible on forums and some software but not on email – the extra “pointless” emails can still be necessary to close the loop.

      1. Loulou*

        How would retracting or editing the question be better than just having both question and answer as a record in people’s inboxes?

        1. A Feast of Fools*

          I’m thinking of “ETA – Info received”, like on social media platforms where the question is in the original post.

          It won’t work with email, though, since the questions and replies are already in people’s Inboxes.

      2. Lyudie*

        Outlook lets you do that, but then everyone gets an email that you want to recall the message so it’s actually not that useful IMO. Also makes things worse if you stuck your foot in your mouth and want to pull the email back.

    2. Antilles*

      I find the quick “thanks” often *prevents* pointless emails. The emailed thanks makes it clear that everything is dealt with, rather than other people also jumping in with their answers.

        1. Loulou*

          Sure, and sometimes the question itself is something that could have been answered by looking at the piece of documentation that the person being thanked referred to. That’s just how it goes.

        2. Reuben*

          Hence, “often”.

          When a comment doesn’t make a universal claim, replying to point out that it is not universal is merely unnecessary chatter.

        3. Rolly*

          I’m certain Antilles included the word “often” because they know it doesn’t always do that.

            1. Antilles*

              I said “often” because in my experience, it commonly is necessary. In the cases where I asked a question and didn’t do a reply-all, I’ve found that someone typically will follow up shortly thereafter or even a few hours later, assuming that I still need advice or suggestions.
              But YMMV I guess, maybe it depends on what kinds of questions people are normally asking and how likely it is that there’ll be variety in the answers. “Where are the truck keys?” only has one answer, while “Does anybody have a good contact at Teapot Co” can easily get 5-10 different people each providing their own contact even if I really only need one name.

              1. Allonge*

                And if a company / department / team is used to getting a reply-all thank you, then of course its absence has a specific meaning (no reponse yet). So this is very much self-perpetuating.

  5. WS*

    #2 – I also work in a job where only people with certain qualifications can answer particular questions (healthcare in my case), and the right thing to do is direct the asker to the appropriate person immediately. If your boss wants you to occasionally do particular work out of your area under their supervision, that’s fine, but it’s literally above your pay grade to be doing this for other people in your firm.

  6. General von Klinkerhoffen*

    LW2, speaking as a paralegal who encounters this same problem, I think it depends on the context of the request. Your letter phrases the request in the passive, so I can’t tell who’s asking you.

    If it’s a client, I think it’s more straightforward: you can simply say something along the lines of, “Sure *we* can do that. I’ll pass it along to (appropriate coworker) and she’ll be in touch with details.” Sometimes clients clock that your hourly rate is lower than an attorney’s, so they see if they can get the job done cheaper. Your comment “However, for most tasks that are above my level/position/pay grade, I probably can work through the request eventually, but it takes significant time” may not be obvious to them in such an instance.

    Alternatively, if it’s someone internal but presumably senior to you doing the asking, it might feel more awkward, but you’d want to fall back on Alison’s usual scripts along the lines of, “Sure, but as I don’t have experience in this area I expect it to take four hours/days and I’ll need to put aside the Klinkerhoffen file in the meantime” or in fact “I think it’s usually Ermintrude who handles this kind of thing – should I pass it to her?” The tone etc of your reply will depend on your office dynamics, of course!

    “I already have an extremely heavy workload, so it is a major strain.”

    Often in law (as in other fields) the best thing for everyone involved is for the least qualified person possible to do a thing. If a trainee can fill out the form for $150/h, don’t get the department lead to do it for $500/h. If it has to be a qualified attorney for indemnity reasons but any literate human could do it, get the attorney who qualified last week to do it. And so on. In some cases it will still be cheaper or better for the firm for you to spend more time on a task than a more expensive fee earner spending less time.

    Also, all this extra self-directed learning and new skills goes on the list for review season, yes?

    1. My dear Wormwood*

      I’m so sad you didn’t continue the naming convention – “Hmm, I think Helga usually handles this.”

      Helga: “SEND IT IN HERE.”

    2. PayGradeOP*

      Question asker here – the issue is 100% with my main attorney, not with clients. With clients, I can let them know I will need to check in with the attorneys (and in fact, I always do that unless I have specific guidance about a particular question – like if in every situation, if X happens, the attorney recommends Y, I can just respond).

      With my main attorney, it is more of a gray area, but I do think just being told I’m overthinking it is a helpful framework! I have been trying a few tactics over the last few weeks to see how the attorney would respond to me pointing out something was a bit above me.

      For example, the attorney forwarded an email expecting me to prepare a draft reply. Instead of trying to figure it out, I responded indicating the question was a bit high level for me, and asked “is there someone else who may know the answers – maybe Helga?” It actually resulted in Helga immediately getting the email forwarded to her, and unsurprisingly, she knew the answers off the top of her head – I would have likely spent hours reviewing prior messages covering similar topics to cobble together an answer.

      I am certainly my own worst enemy here, too. I am definitely an obliger, and I like to approach issues with, how can I do this? – rather than immediately thinking nope, I can’t do this. It can be a double-edged sword, and it is good to hear that I’m overthinking things. I don’t want to disappoint my boss – but likewise, because there’s so much I *could* answer quickly that many at my level couldn’t do, he might be assuming some tasks are simpler for me than what they really are.

      The script suggested is a great idea, too, and I’ll try it out when this inevitably comes up again! It feels a lot less abrupt than flat-out saying that’s above my pay grade, even if that’s the spirit of the message.

      1. All Het Up About It*

        Does it help if you think about tagging in Helga as a method of how you can do something?
        I appreciate the approach of “how can I do this,” but if you encourage yourself to think of delegating/directing to the correct person as exactly how you can do something, it might help. You getting something done does not always mean that you have to be the person to do it.

        1. PayGradeOP*

          That’s a good suggestion, but hierarchy does get in the way a little bit. I work for an attorney – the attorney can delegate work to me or to associates, but I can’t delegate work to associates. That’s why I need to go back and ask could Helga do this?

          I love though: “You getting something done does not always mean that you have to be the person to do it.”

          That is AWESOME framework and I will definitely be ruminating on that a lot!!

      2. Purple Cat*

        Super important context if it’s “your” attorney sending you the work.
        A specific conversation with them as to how they want you to handle this going forward. They may *want* you to have the development opportunities, or may simply be blind to the fact that they’ve put something too advanced on your plate. Setting the boundaries/expectations is really important.

        1. pancakes*

          If they’re like the more senior attorneys I’ve worked with, they’re simply expecting things they delegate to continue to be appropriately delegated from there. But yes, definitely worth a conversation.

      3. The OTHER Other*

        OP, I get how this work is taking you away from regular duties and adding to your work load. But where you are seeing burden, I am seeing opportunity.

        Your bosses are trusting your judgment and ability on a far higher level than your peers; this is a good thing. It sounds as though this is higher-level work. To me these are good arguments for a title change and salary increase!

        Of course, this presumes that you would WANT to do this work versus your ordinary duties. And that your workplace is not the sort of place that wants people to do higher level work without paying them for it.

        1. Kit*

          In some workplaces, this might be true – in this one, where the higher-level work is the sort that is usually handled by attorneys (who have both a legal education and an accreditation that the OP lacks) there are fixed hurdles to certain promotions, and while OP’s knowledge base and capabilities are well worth retention, it’s not an optimal use of her time to attempt to take on this work when another attorney could handle it much more quickly.

          OP also mentioned above that one of the obstacles is that because she is not an attorney, she can’t technically delegate such work – that’s why she has to go back to her primary attorney to ask him to delegate it to one of the associates! There’s certainly a degree of trust, but these requests are a waste of both his assistant’s time and his own, since OP’s regular duties are delayed every time she attempts to handle this work. It’s much faster and more effective for everyone if she simply suggests that he pass it on to one of the associates in the first place.

      4. Another JD*

        Attorney here. There’s nothing wrong with acknowledging you’re not the best person for the job. It’s important for your partner to know when an assignment will take you a long time and interfere with other priorities. I’d be pissed to see a paralegal took 5 hours to answer something I could have done in 30 minutes if they didn’t talk to me about it first.

        Sample script: “SEC issues are outside my wheelhouse. I could probably figure out an answer, but it would take 4-5 hours and I’d have to bump the Smith interrogatories until tomorrow. Do you want me to get started, or should I see if SEC Helga has time to take a look?”

        1. PayGradeOP*

          I tried this and it worked out really well. the attorney read my message and forwarded it to the person appropriate to do the work – which was helpful because I’m still in the loop to answer basic questions about the case, but I don’t need to do the high level task.

          (This is now INCREDIBLY important because a colleague went out on leave indefinitely and I got all of the colleague’s cases… yikes!)

  7. Aximili*

    3: Am I… not supposed to be thanking people? When I ask a client to send me something and they do, I’m pretty sure a “Thank you” is the right response. For all the others on the email, this means they know that I got what I need and no further action is required. It’s polite and makes good business sense.

    1. Laure001*

      Of course! What the OP is saying is that you should not use “reply all” for your thank yous, just “reply” to the person you’re speaking to and not burden the others.
      But I disagree, actually, for many reasons stated by other commenters, and you make another good point. If you are in communication with a client and the client and your manager are cc ed, it’s important that they saw you wrote a polite thank you mail. If you don’t “reply all”, your manager may not see your thanks and think you were rude to the client, for instance.

      1. Allonge*

        I see why that could be an issue but – that’s a really crappy manager, then. I don’t need to see every move of my employees to think they are appropriately handling client communication.

        1. QuickerBooks*

          Sometimes you’d like to know that a particular interaction was handled to everyone’s satisfaction. That does not make someone “a really crappy manager”.

          1. Allonge*

            Sometimes? Of course not. Managers have to have a way to quality control.

            But if in a single instance a manager does not see a thank you email and from this they go to ‘my staff member was rude’, they are a pretty crappy manager. If the only system they can come up with is to be cc-d in everything, that too is suboptimal.

            1. A Feast of Fools*

              But it wouldn’t be just a single instance if the staff only ever replies directly to the sender. Every single time the manager was cc’d on something, she’d never see the staff person doing the right thing.

              1. Allonge*

                And, again, a manager needs to have a way to deal with that without being cc-d in on everything. Make it part of the onboarding. Have a template. Ask staff if they are doing it. Ask clients if they are happy with how they are treated. If there is an issue, address that.

        2. ecnaseener*

          When the entire conversation has been reply-all up to that point, people naturally assume it will continue to include them because it was a group conversation. It’s nice that you’re not bothered, but I don’t think it’s only bad managers who would operate on that assumption.

          Incidentally, that’s why switching to a private reply just for a “thanks” feels weird to me — it’s like deliberately pulling someone aside from the group.

          1. Antilles*

            Especially if every previous email included everyone; if you reply only to the client, the natural assumption by others is that nobody’s said thanks yet so someone will usually jump in to close that loop.
            And that’s not necessarily because of a lack of trust. In fact, it might be because they DO trust you – basically that I’m replying on your behalf because I’m assuming that you just didn’t see the email yet since you’re juggling lots of projects.

          2. L.H. Puttgrass*

            ‘Incidentally, that’s why switching to a private reply just for a “thanks” feels weird to me — it’s like deliberately pulling someone aside from the group.’

            Exactly.

            And I can’t even count the number of times a “closed” loop has reopened. Not long after the “Thanks” e-mail: “You’re welcome! Oh, one clarification….” or, “By the way, on this other topic…” If the thank-you e-mail just goes to one person, the e-mail thread has been split and people need to remember to add the others back in. It can get kind of messy.

            1. Smithy*

              This here is why I would never “close the loop” with a direct reply thank you.

              At least in my work, there can be a lot of soft closures. You send someone a proposal that you’ve been back and forth on for a while. In the final submission go, I’ll usually include a line to reach out in case they have any more questions or require anything else. It’s not out of the norm to get one email that says “Thank you, we have everything we need now”…..and then in a few days or hours to get another email “Finalizing the Objectives tables, and……”

      2. Snow Globe*

        When I just “reply” rather than “reply all” what often happens is that my manager or others will soon “reply all” their own thanks, which then makes it look like I am the only one that didn’t send a thanks. Ugh.

      3. Just Your Everyday Crone*

        This may depend on the kind of work that people do. I manage lawyers and needing to see that they handled something politely would be an absurd level of micromanagement.

      4. kittymommy*

        Depending who’s on the email chain I sometimes cc all and sometimes not. Many times if I am making the request at the behest of my boss (highest level in our org) and the colleague and the colleagues boss(es) or on the chain I will cc everyone as to acknowledge to the upper level people this person and their assistance. I want to make sure they get the credit.

        “everyone” – 4-5 people, btw.

      5. Unaccountably*

        If you and your manager are the only ones cc’d, that’s a different issue than having your “Thanks, Jane!” go to twenty people who were probably only tangentially involved in the email issue anyway. There’s room here to use common sense.

    2. anonymous73*

      If I see that client sent you what you asked for, I will assume that you got what you needed and will respond again if you didn’t. In your example a “thank you” response is good for the client (you’re confirming you received it), but unnecessary for the others. Yes it’s a minor annoyance, but still an annoyance and yet another interruption in my day.

      1. Esmeralda*

        I hope you leave a decent amount of time before jumping in to help.

        Because that’s another way reply-alls multiply, AND then it looks like I didn’t do my job. Now I’m annoyed at all the reply-all AND I’m peeved at whoever didn’t just let me do my job.

    3. theletter*

      I think it’s more a matter of knowing when to close the loop, and when it’s assumed that messaged received is message confirmed.

      ie, I had a manager who would reply ‘thank you’ to emails reminding an internal team when a teammate would be OOO, and other things where I just really didn’t need to know that Bob knows that Joe will be out. It got to the point where we’d put EOM on certain emails, only for that to be commented on in a reply-all email.

      There are absolutely times where letting a client or stakeholder know that a file has been received, an update is understood, a time sensitive thing has been resolved, etc etc, that a thank you to everyone involved really fits.

  8. RagingADHD*

    LW2, I think this advice really misses the context of how law firm hierachies and task assignments usually work. Clients, members of the general public, or members of other departments don’t request staff members to do legal work.

    Clients especially don’t want non lawyers doing their legal work!

    Certainly, if it’s a partner or senior partner choosing to delegate this to you rather than to an associate, you absolutely should not say “we usually give this to Cecil.”

    There is no “we” in that scenario. The boss is telling you, because they want Cecil doing something else.

    The issue I suggest you are better off raising, is how to balance your workload. When you’re given a time consuming assignment, ask which of your other deliverables should get bumped. If you would be willing to do overtime, you could ask if the task is authorized for overtime. Depending on the situation, the boss may feel that this new task is a higher priority, and it’s fine for something else to be delayed.

    One of the good things about a very hierarchical environment is that there is a clear line of whose projects get top priority, and who needs to ask permission to bump a deadline.

    1. pinot*

      the letter doesn’t say it’s a law firm though and it could be all sorts of organizations that have teams with attorneys on them. i read it as the requests coming from members of other depts though on 2nd read it’s not necessarily clear.

      1. Just Your Everyday Crone*

        I am in that sort of an organization and we don’t have clients making requests of the paralegals.

        1. pancakes*

          Of course not, but I’m not sure where anyone is getting the idea that these requests are coming from clients. It sounds to me like they’re coming from other internal departments — communications or investor relations, for example.

          1. RagingADHD*

            I listed clients as one example of people who were not the supervisor, and might need an explanation about how the workflow works.

            Someone outside the hierarchy might *request* a task, but the correct response is always to check with the supervising attorney. A staff member should not be proceeding with legal work at the direction of a another non-lawyer.

            The point being that nobody authorized to assign the work needs to be told how “we” usually do things.

      2. RagingADHD*

        1) In general, the term “attorney” is most often used to describe the members of a law practice. Lawyers working in-house at another sort of company are more often referred to as lawyers, counsel, or the legal team.

        2) Even when I supported in-house counsel, staff members of the legal department did not accept assignments of legal work from anyone other than a supervising lawyer. That’s part of the relationship that makes it kosher to delegate the work. The lawyer has to decide what can be delegated.

        For a non-lawyer to accept that assignment from anyone else, without at least checking in with the supervisor, could infringe the limits of practicing without a license. Some places might establish some kind of “standing order” to always take a particular task to the LW, but that’s….not good practice, according to the way I was trained.

    2. Willis*

      Yeah, I spent some time doing paralegal work and agree with this. If it’s a question from a client, I’d check with my boss and see if they want to answer, want me to answer, or want to forward the question to someone other associate. But if they’re requests from coming from attorneys on staff – and definitely from a partner or senior partner – I’d focus on this as more of a workload issue than anything else. The OP has the same title as others but it sounds like more seniority/experience (so possibly higher pay?)…there may be legitimate reasons she’s being asked to do this stuff that would make it weird to try to redirect the work. This sounds like a call her boss should be making, not Alison.

    3. lizesq*

      Yep! If it was at my firm and a paralegal/legal assistant pushed back on doing say, title or survey review that an attorney asked them to work on, it would be really odd and would not go over well at all! It’s about knowing your workplace but, at a lot of firms, Alison’s scripts would not be ok.

    4. General von Klinkerhoffen*

      You phrased this much better than I did. Often there may be regulatory and/or insurance/indemnity reasons for having a particular job title handle something, but there’s also complicated dynamics about what can be charged at what rate, ie whose time is more valuable and whose time is more expensive.

    5. NotAnotherManager!*

      Many clients have no problem with non-lawyers doing their legal work. We had corporate clients who flat-out would not pay for attorney time to do what they considered to be lower-level tasks (form motions preparation, basic document review/priv logs, witness binder prep, etc.) or they would only agree to pay a rate for a paralegal regardless of who did the work. Some outside counsel guidelines explicitly state that works should be done by the lowest-billing personnel with the capacity to competently handle the work.

      I worked with a number of senior paralegals who knew more than most first through third year associates, at least. I would have trusted one of them with legal matter more than an associate with less experience. They could not sign things, but our clients often had a relationship with them and would ask to have them do their work because it was a better value to pay their rate for the level of expertise they had over most of the associates. And they’ve know some of them for well over a decade.

  9. Roxanne Melman*

    LW #2, as a long time professional in the legal world, but not an attorney, I’m intimately familiar with this. It even comes up in my annual reviews along the lines of “you’re doing first year associate-level work”. Yes, it’s a pain to answer interrogatories and request for production. Bottom line for me was always if I’m being asked to do a particular task, even a very time consuming task, I had to assume that the boss had enough confidence in me and my abilities to trust me to complete the task … or at the very least get the first draft down on paper for the attorney to review. It’s not about “an attorney should be doing this” but rather it’s about having someone who bills at a much lower rate do the nug work and get the first draft on paper for the attorney to review. This frees up the attorney’s time to handle matters (billed at a much higher rate) that I cannot (legally) handle because I’m not an attorney, or be in meetings or on phone calls with a client discussing the case (which is not something I’ll ever do beyond calling a client and asking a question on behalf of the attorney). Bashing out a first draft of discovery responses is a common task for legal assistants and paralegals. It saves the firm money and it saves the attorney’s time for me to do it, and it adds to my skill set. The more you do, the easier they get, and the more you learn, all of which results in you becoming invaluable to your attorney/law firm. But! When you get assigned to answer discovery, it’s really important that you (a) manage time expectations on how long this will take you (meaning you must make it clear to the boss, “I’m going to start responding to discovery in the Smith case and this will take a lot of my time. Are there any other tasks you need completed first before I get started?” Or “I’m going to start on the Smith discovery responses and if you need to interrupt me for a more important task while I’m doing that, please understand that it will result in me taking longer to complete the discovery and is that okay?”) (or something along those lines) and (b) you need to manage any anxiety that crops up about not getting other stuff done – perhaps the stuff that you are better at or enjoy doing more (this is my personal issue – may not be yours). At least that’s been my experience after 20+ years. But if you really are uncomfortable operating at that level or feel unqualified to perform that task, I recommend having a serious sit down with your attorney-boss and explain why you aren’t comfortable/don’t feel qualified doing this task. Good luck!

    1. MK*

      If someone is doing work usually done by associates, I would disagree that there is no unauthorized practice of law. I assume that having an attorney supervise means there will be no disciplinary consequences if someone found out, but it’s a pretty blatant violation of the spirit and intention of the rule.

      1. Emmy Noether*

        Not necessarily. For example, I had a job where certain forms were filled out by assistants, then someone with the required qualifications would check that it’s correctly filled out and sign the form. The signatory guarantees with their name that it’s correct and for that they need the qualifications. They do not have to physically fill out the form themselves though.
        Same works for other tasks. The attorney/whatever vouches for the accuracy, but they do not have to actually have written the thing themselves. We also had a lot of letters and texts written by people still in training, and then reviewed and signed by someone qualified. That’s fairly normal.

      2. Roxanne Melman*

        Not practicing law. Answering questions to get a first draft done and get the formatting set up for the entire document so that an attorney can easily make changes. The first draft never goes out the door. It’s reviewed and changed and reviewed and changed again by boss and maybe even grandboss. But the review takes less time and is billed at a higher rate than the draft, which takes longer and is billed at a lower rate. If the paralegal isn’t versed in this type of task then sure, the work is given to a first year associate so they can learn how to answer discovery. But it’s cheaper for the first draft to be done by a paralegal/legal assistant.

      3. Forrest*

        The spirit and intention of the rule is that there is someone who can be held legally accountable if the form or decision is wrong, and that’s the difference in status between a attorney and a paralegal or similar role. The spirit of the rule isn’t “this work should be done by an attorney”, it’s “whoever did the work, we need the name of an attorney who can be held accountable by their professional regulator if it is wrong”.

        Same with other regulated professions– it doesn’t matter whether B G Gruff CEng is the person who did all the calculations for the bridge and stiplated what screws to use. It matters that if the bridge falls down, B G Gruff’s name is on the bridge design and she can be held accountable by law. She has the authority to delegate parts of that work, but also the responsibility for making sure it’s done right.

      4. Richard Hershberger*

        “Doing work usually done by associates” is a pretty decent first approximation of the job description of a paralegal. That job title is a fairly recent development, in the big picture. The whole point of the position is to do stuff more cheaply that otherwise an attorney would have to do. What stuff? It depends on the paralegal. A junior one will be doing routine grunt work. An experienced one might be doing research and writing motions. This is not practicing law because it is still an attorney who will be signing that motion (or authorizing the paralegal to sign their name to it), presumably after reviewing it.

        What I cannot do as a paralegal is give legal advice, sign my own name to pleadings, and suit up and go represent a client in front of a judge. Beyond that, it is a question what I am able to take of my attorney’s plate.

        In my years doing this, there has been one occasion where I almost signed by boss’s name to a pleading without his express permission. It was a response to the other side’s (bogus) emergency motion that needed to be filed that day, and my boss was in court. I managed to get ahold of him on a break and read the response to him over the phone and got authorization to sign it. I would have filed it anyway, for this boss. I have had others I would not have. As for talking to judges, I have done this on two occasions, both times on errands to the courthouse where I expected to be talking only to clerks. In both cases I opened things by explaining that I am not a lawyer and why I was there. This is how people like me stay out of trouble.

      5. NotAnotherManager!*

        There is no single standard for “work usually done by associates”. It varies by firm and by the available non-attorney personnel. There are obvious lines like being able to sign pleadings, taking a deposition, arguing a motion in court, or even becoming a member of a bar or court, but drafting pleadings, filling out endless forms or form pleadings, and doing discovery-related tasks are perfectly fine. Attorneys have a duty to supervise those work for for them, and they are on the hook for anything submitted under their name, whether they drafted it or not. Outside of that, they have a lot of leeway on how work gets done.

    2. Jessica*

      What is nug work? I understand your meaning in context, but I don’t know this expression.

        1. How About That*

          Pet peeve: use of miltary terms in non-military settings. Things like signing off with “v/r”. Does not enhance communication.

    3. Karia*

      It’s not an anxiety issue. It’s a getting in trouble because her core duties are not completed issue.

      1. Roxanne Melman*

        I re-read the letter. Nowhere could I find the OP fearing she would get in trouble. Instead I read: “ I already have an extremely heavy workload, so it is a major strain.” That sounded like some anxiety to me. Honestly her core duties are whatever she’s assigned to do. If the boss/attorney needs her to answer questions that are time consuming, the boss/attorney knows her focus will be taken away from her other duties, perhaps even for a substantial chunk of time. The boss is willing to sacrifice those other duties for the time being in the interest of getting the questions answered. Based on my paralegal experience, it sounds like the boss/attorney wants her to learn this skill (though I could be way off here) and my experience is that the more you answer questions like this, the easier it gets and the faster they go with each new set of questions. It also sounds a bit like she just doesn’t want to do that part of her job that’s been assigned to her by boss/attorney because it is so time consuming and is hoping to offload that task to someone else. Not really how it works in any law firm I’ve worked in. (Again, this is supposition on my part). But you’re right – she did not specifically say she had anxiety.

  10. Laure001*

    I think it’s important to “reply all” the thank you, so you know the conversation is closed, and you don’t research the question yourself to give a more complete answer than the one that’s already been given.

    It’s also a way to give kudos publicly to the person who helped you in a timely manner. The public thanks say, “Rachel showed competence in giving the right answer when I needed it.” So you know you can ask Rachel next time if you are stuck on something: she’s the right person, with knowledge on this subject.
    And because you know her answer was useful, you won’t be surprised when she’s promoted.
    In short I think the public thank you is very useful!

    1. Smithy*

      I do think that the second part of your reply has value – this goes to all of the “how are people motivated at work”. And while lots of people aren’t necessarily motivated by this kind of gratitude, this types of thank you’s do often help build generally good working connections at work.

      However, the first piece I think is why the email needs to go out. Essentially just to say “got it, no more work needed” – and instead of writing it that way, saying “thank you, this is exactly what I need” conveys the same information in a more thoughtful and kind manner.

  11. Princess Xena*

    #4 reminds me of something that happened at the public accounting firm I work at. In the area I’m in, there are two internships – summer and winter. Summer is more of an intro to accounting internship. There are more activities and get to know you’s, and the students accepted are often just getting started in accounting. This can result in interns coming in who have not yet taken audit classes in college, which is usually the class where basic accounting ethics is taught.

    The end result of this is that an intern working on a client (with privileged, unaudited, nonpublic information) mentioned to their manager “hey, this client is doing really well, I should buy their stock”. There followed a brief but very urgent lesson on the definition of insider trading and what not to do.

    So for heavens sake, if you’re getting interns who are unfamiliar with the practical aspects of ethics, client confidentiality, and privileged information, tell them what not to do, because not all colleges cover material at the same time and no one wants to be the intern who got into trouble by accident.

    1. Wildcat*

      It’s never, ever a bad idea to have an ethics refresher but what makes this particularly bad is even a 3L at this point should have taken legal ethics and the MPRE (a separate standardized ethics exam you usually take after 2L) Both the fact that you shouldn’t disclose client information and the fact that LW’s firm is required to report this breach are explicitly covered in that exam (and the prep class you often take) and the required Legal Ethics class.

      So I absolutely definitely agree that it’s never, ever a bad idea to have refreshers on legal ethics (particularly as some states have an ethics requirement for yearly continuing legal education hours), thile applicant here absolutely should have know better. It’s literally part of a required test and exam.

      1. NoviceManagerGuy*

        There’s also a common sense issue here. If you need an ethics class to know you can’t send out client information in a job application, you don’t have the basic good sense to be trusted with client information.

        1. The OTHER Other*

          The regs in my industry can be arcane and also change frequently. These are often covered under the topic of “ethics” courses, but yes the basics–don’t lie, don’t cheat, don’t steal, don’t freaking send confidential client data TO THE OPPOSING COUNSEL should be obvious. Yikes.

        2. Elysian*

          This is so tricky for new lawyers, though – the industry doesn’t help. Nearly all jobs require a writing sample, and pretty much all writing that new lawyers do is confidential and privileged client information. Every job wants a “real” writing sample, not a contrived one that you wrote just for the purpose of the application. But, you can’t always ask permission from your current employer to let you use a document as a sample, because then they’ll know you are job searching. It is an awkward mess to find a good sample. You’re stuck looking at all your writing trying to find something that is both good enough and public enough. Sometimes even redacting it isn’t enough to keep the info confidential!

          So this applicant made a mess of things (obviously, sending the document to opposing counsel is the absolute WORST type of ethical violation), but I feel some sympathy because the application system just doesn’t set people up for success when it comes to writing samples.

          1. pancakes*

            It isn’t great, no, but that’s the sort of question law school career counseling centers should be able to field, and even a 1L should know they’re better off asking someone with more expertise than winging it.

            1. Elysian*

              It seems like a lot of people had more active law school career counseling people than I did! After I graduated I would never have considered reaching back out to my law school career folks. They were good for OCI and internships and that was about it.

      2. TiredMama*

        I’m pretty sure we covered this in legal writing 1L year when we talked about legal memos and when to add ‘privileged’. I think we talked about it again in a variety of class from civpro to evidence – privilege, evidence, objections, etc.

    2. Richard Hershberger*

      There at least the intern has the excuse of nothing yet having been taught professional ethics. In this letter it is an actual attorney: Wow. Just wow.

      1. quill*

        The only way to make it better is to presume it was the wrong document, but holy forking shirtballs you should check your attachments and maybe not HAVE that unredacted document to begin with.

        1. JSPA*

          I immediately assumed that they were either looking for an “in” by offering to share “useful” information, illegally…

          or that they had an axe to grind and were looking for even a threadbare excuse to injure their current employer…

          or that they were doing it as some form of whistleblowing, because they had qualms about the person or group that their firm was representing.

          Note that any of the above render them not only un-hireable but quite probably on the wrong side of several laws! But people get strange ideas from television, film, and their friend groups. The understanding that sending info to opposing council is not just the law firm equivalent of spelling out “I QUIT” in fish, may have eluded them.

    3. Katie*

      I do accounting as well and this letter made me cringe a lot. My company takes confidenality seriously and we have training courses all the time about how even internally how this is not good.
      We noticed recently a piece of confidential info on an internal email and the work to scrub that email was so time consuming and I was so frustrated with the person who sent it out because it was never okay.

    4. Cj*

      My FanFiction explanation is that the applicant did this on purpose. She had false information in the document that would cause the OP firm to lose the case when they assumed it was accurate.

    5. Artemesia*

      This is the sort of training that any new intern in an accounting or law firm should be getting. A quick lesson in privileged information, confidentiality and insider training is a minimum before they work a single day.

    6. Nope, not today*

      I finished up my accounting degree about… four years ago? And in my first few years of school – back in the early 2000s – there wasnt a whole lot of ethics discussion outside of my audit class. But this time around every single class had an ethics portion, accounting classes and business classes. We must have discussed Enron and SOX in a dozen different classes – it was quite a different experience from my first round of accounting courses! Ethics has become recognized as being SO important that its a huge focus, so I would imagine even new students would have had some exposure to it if they’ve taken any accounting courses at all.

      1. Princess Xena*

        I found it equally bizarre, but I was already interested in learning about fraud and true crime before I ever went to college so the idea of not understanding what insider trading consisted of was and is super bizarre to me. I think it only happened because the summer interns at my firm are often recruiting from rising juniors and seniors who have a very mixed class background.

        But it also seems to have been rare enough that my firm never thought about having to have “no insider trading” lessons during orientation, so maybe it was someone who was just unusually uninformed.

  12. I'm the Phoebe in Any Group*

    1, I agree with Alison, but would go further with this sexist community member. I would call out his interruptions each and every time so that interrupting you becomes less pleasant for him. I would also be decreasingly polite with subsequent interruptions. As a goverment employee, you might have less leeway than in business, but you can also use tone of voice and a hard glare: “Floyd, as I was saying before you interrupted me,…” Call him out on exactly what he is doing: “Floyd, I am the project manager and am the person with the information you want. Why do you keep requesting that a man answer instead of me?” Bonus points if he sputters during his denial. If he mainsplains your field of expertise to you, “Floyd, I am expert on traffic light timing in urban thoroughfares and have two advanced degrees in the field. The reason we stagger red lights during rush hour by 17.85 seconds is…”

    1. Forrest*

      I’m not entirely clear on the kind of meetings these are, but if they are to engage the community in project development and implementation then this kind of credential-waving may not be appropriate. You can’t simultaneously be trying to engage community members by telling them their lived experience is valuable AND wave your formal education to establish a hierarchy! You might need something much more subtle.

      LW1, I think one thing you and your manager should do is sit down and get really clear in your head what the hierarchy is at these meetings. Do you want community members to be in the driving seat? Does anyone chair the meetings? If you don’t have a chair, would they work better if someone did chair them, and is there anyone from the community (other than Mr Interrupty Pants) who you could ask to do this more formally?

      If it IS apppropriate for you to cite your credentials to shut him down, then by all means do that! But the hierarchies and sensitivities at these kind of meetings can be pretty complex, so you don’t want to do that if it’s not going to help your organisation meet yours (and the community’s) goals.

      1. I'm the Phoebe in Any Group*

        Allowing a man to talk over and ignore a woman so he can speak to a man instead needs to be shut down

        1. Forrest*

          I think it really depends on what the goals of the meeting are! Being super cynical, if it’s to allow community members like this to feel like they’ve Had Their Say, even if that is then ignored, then LW might have to swallow it. If it’s genuinely to allow people in the community to contribute to the project, then it’s important to manage this community member but in a way which doesn’t suggest that the people with degrees are the most entitled to speak. If this is a community where there isn’t a big culture of professional accreditation and expertise, you need to be really careful about the power relationships and make sure that the way you shut this particularly obnoxious person down doesn’t dissuade others from participating by establishing a hierarchy where “more degrees=more entitled”.

          1. I'm the Phoebe in Any Group*

            Women having a right to speak is not a situational thing. This rude, sexist guy asks a question, OP starts to answer, then he talks right over her and says he wants someone with a penis to talk instead.
            I don’t know your gender, but if you are male, you need to stop telling women to accept being shut down.

            1. Forrest*

              I’m female, and I think it’s really important to be aware of power dynamics at play as well as gender. If LW and her colleagues are in the room as credentialled professionals, and the community members are not, that is also a power dynamic which needs to be understood and accounted for.

              1. Loulou*

                +1000. I get the righteous indignation, but this is very possibly a more delicate/nuanced situation than some commenters seem to appreciate. It is likely not appropriate to deal with it the same way you’d deal with a colleague with the same credentials as you acting this way.

                1. How About That*

                  Exchange sexism for racism and see if you would feel the same way. It would never be acceptable to always shut down the black person and ask for a white person to respond, community meeting or not.

                2. Forrest*

                  Literally nobody is saying it’s acceptable! C’mon. There’s a big difference between “you might need to find a way of managing this guy which isn’t ~shut him down with maximum prejudice~ if this is a situation where you need to be careful to preserve the good will and trust of the other community members” and “he’s cool, just suck it up.”

                3. Spencer Hastings*

                  Nobody is saying that what the community member is doing (constantly shutting down the LW) is OK. But there are better and worse ways to deal with it, just tactically speaking. And that would be true regardless of whether the issue was sex, race, or…heck, even if all the colleagues were the same sex/race/age/etc. and the community member was inexplicably dismissing one of them.

                4. Loulou*

                  How about that, since you brought up racism, now why don’t you imagine that OP is white and middle class and the community member is not and see if that helps you envision the potential dynamic Forrest and I are talking about?

              2. Starbuck*

                And if I’m a woman community member in the crowd with an opposing viewpoint but get too intimidated on seeing this is how the meetings are managed to speak up – how is that serving the public good? (And this is not a hypothetical scenario, it’s happened in my own local govt). Failing to manage the recurring bad behavior of certain residents can make other people less willing to speak up who deserve to have their input heard too.

                1. Forrest*

                  But nobody is saying *don’t* manage it! We’re saying that depending on the community LW and her team are working with, it might need to be more delicate than “smack him down with your credentials”.

                  I can’t believe this needs saying, but not every woman gets a kick and feels empowered by seeing a professional woman slap down a man in her community. Some of us— even some of us *with* credentials!— don’t find that empowering or positive.

            2. ecnaseener*

              Forrest isn’t saying to accept it, just to deal with it in a way other than declaring your credentials.

          2. Esmeralda*

            He can have his say, but OP’s manager and colleagues should have her back and redirect this bozo every time. Politely, of course. Alison’s scripts are good. OP needs to ask her manager and colleagues to do this, if they are not already doing so. (And if they’re not doing so, shame on them, but no surprise alas)

          3. Worldwalker*

            The issue is not whether the OP is more entitled to speak than the community member, but whether she’s more qualified to answer than a co-worker.

            This guy doesn’t say “Fred, what do you think about X” — he waits for the OP to start answering, then shuts her down.

            He knows exactly what he’s doing.

            1. cynara c*

              Exactly. This is not OP quashing the guy’s ability to provide feedback or share his experience. It’s the guy actively undermining the expertise of a woman who is likely the best person in the room to either answer the questions or to delegate who from her team can best provide answers.

          4. Bee*

            The goal of the meeting is to hear from the public, and the people running that meeting should ensure that the environment is inclusive. If they allow a community member to shut down a speaker because she is a woman (or part of any other marginalised group) this will also signal to other women that their input is not valuable. So no, LW1 shouldn’t just suck it up.

            1. Forrest*

              You’re responding to literally the OPPOSITE of what I said!

              If it’s genuinely to allow people in the community to contribute to the project, then it’s important to manage this community member but in a way which doesn’t suggest that the people with degrees are the most entitled to speak. If this is a community where there isn’t a big culture of professional accreditation and expertise, you need to … make sure that the way you shut this particularly obnoxious person down doesn’t dissuade others from participating by establishing a hierarchy where “more degrees=more entitled”.

              Like, I don’t know how to put this more obviously, but I literally wrote that if this is a genuine meeting where the community voice is valued, “you need to make sure that the way you shut this particularly obnoxious person down doesn’t dissuade others from participation”. Not that if the goal is to hear from the public then LW should suck it up!

          5. ElizabethJane*

            Except the issue isn’t that the LW doesn’t want the community member to contribute it’s that she wants the community member to let her answer the questions he is asking.

            He’s still 100% allowed to contribute, to engage in polite discussion, and to ask whatever questions he wants to ask. He’s not allowed to dictate that only men answer him.

            This isn’t a matter of community engagement. This is just a man being a turd.

      2. Myrin*

        I think that’s coming at Phoebe’s suggestion from an unnecessary angle. OP would be “establishing a hierarchy” over her lower-ranking male colleagues, not the community members – she is, in fact, the project leader and it’s okay for her to point that out, especially in the face of blatantly sexist (not to mention plain rude!) behaviour like the one described.

        I’m pretty sure I know the kind of meetings OP is talking about – we have them pretty regularly in my municipality/county – and it’s one thing to talk over the attending community members in a way that leaves them going “Okay, and why exactly did we have the opportunity to come here if those in charge ignore everything we say anyway?” and quite another to call one of those members out on their specific inappropriate behaviour.

        1. Forrest*

          I think “I am a project manager so I can answer this” would be appropriate in any situation. It’s “I have two advanced degrees in the field” which wouldn’t be appropriate in all situations. But I’m just raising it as a possibility– LW knows best what kind of meeting it is and what the relationships in play are.

          1. Myrin*

            That’s true, but I think you’re taking too literally something that was just one of several example sentences where the intended takeaway was the overall message/tone of these sentences, not any particular detail OP would probably need to change to fit her situation anway.

        2. Emmy Noether*

          This. Plus there may be female community members who may actually feel more comfortable contributing if they see sexist jerks being shut down.

        3. kittymommy*

          I said this elsewhere but the highest ranking staff at the meeting needs to take back control of these meetings. It may be a community meeting where the public and government staff can engage in discussion but the staff leads the meeting. It sounds like they have acquiesced a lot of their control to the community, and it’ll likely be a bit of a struggle to take it back, but they need to do it. What I have seen work for us is having all questions be initially answered by the team lead and then verbally assigned out to staff.
          Community member: “Bob, what if we have this issue….?”
          LW/Team lead:” Great question, Paul/Lindsay/whoever, this is more in your wheelhouse, would you mind answering that.”

          Rinse and repeat. When the citizen pushes back the LW/team lead gently pushes back with a “I think is the best qualified person for that subject, but if you have anything further about it you may speak with me after the session.”

          There is a certain type of citizen that needs to smacked with a velvet glove in order to get the message.

      3. Hi there*

        All I can see is Leslie Knope’s first town hall meeting! :)

        OP: I agree that a conversation with your manager is the place to start. This guy won’t get far if there is a united front.

    2. Trawna*

      I’ve led community groups and chaired many community meetings, “That guy” typically shows up, talks a lot, but doesn’t ever do any actual damn work. Politically, you could win community allies by gently, firmly shutting him down.

      1. NoviceManagerGuy*

        Agreed, That Guy showing up has already made up his mind and is just wasting everybody’s time.

    3. anonymous73*

      I said something similar in my comment, but your language is a bit combative. A simple “please don’t interrupt me” will get your point across.

    4. JSPA*

      The experts with credentials keep setting the timing of a traffic light at a complex 5 way intersection near my house to give cars an arrow directing them into pedestrians who concurrently get a walk signal…where the intersection is partially blind. There have been 50 people public protests, but the modeling says traffic flows better that way (blood and glass on the asphalt be damned). I’ve seen plenty of credentials waved. If you’re the person whose family member was hospitalized, or a driver who nearly hit someone, those credentials only add up to, “so that means you should know better, and still don’t.”

    5. English Teacher*

      Yeah, I was surprised that no part of the advice was to just ask the community member WHY they prefer to hear from others. Yes, it is probably sexism or another weird personal thing, but why not just ask in good faith? It’s possible that he has a misunderstanding of the hierarchy, or some other misconception that can be cleared up fairly easily.

  13. I'm the Phoebe in Any Group*

    2, do the requests come from clients or colleauges? If colleagues are they higher on the food chain, lower or relatively equivalent?

  14. I'm the Phoebe in Any Group*

    #4, do you also need to let opposing counsel know about this? You’re the attorney, not me, but I wonder.
    This is a such extraordinarily bad judgment by the applicant. Even all of us non-attorneys know that.

    1. Peachtree*

      I *think* the question is whether – before they tell opposing counsel – they should also give the applicant a heads-up. I.e. they will definitely tell opposing counsel, but should they also inform the applicant first?

      1. Wildcat*

        The applicant is almost certainly going to hear about it from the former firm, it’s probably kind to give them a heads up first. But there’s absolutely no way this situation isn’t going to be unpleasant. This is a huge error.

        In situations where you get privileged information, you’re actually even supposed to stop reading as soon as you figure out what you’ve receieved. They certainly can’t assess it as a writing sample.

        1. RecoveringSWO*

          I think it would be wise to contact the applicant first as well. We were coached that we could submit writing samples from summer work if we received permission from our employer and either redacted or changed names and other protected information. Our counselors actually recommended that we change information instead of just redacting to improve readability of the sample. That said, we also put a blurb above our samples that explained that changes were made and permission was granted to use the sample.
          I think OP more than likely has a correct understanding of the situation. But calling the applicant first would allow them to say “oh no, I forgot the explanatory blurb! I’ve changed details in lieu of redacting and I have permission.” Then, you know, I would still mention it to the other firm during a reference call just in case.

            1. OhNo*

              I suppose it is possible that identifying names/details were changed and the situation is so unique that OP recognized it anyway… but that seems unlikely.

              Regardless, it would be a kindness to notify the applicant first. Bonus points if you tell them exactly when you are going to let opposing counsel know that you got it – that way the unfortunate applicant can have a preemptive “I messed up, you’re about to get a call from your opposition, here’s why…” conversation with the relevant parties.

          1. Loulou*

            I think we know this can’t be the situation this time because OP recognized the case as one that they’re working on, but this is an interesting point I hadn’t considered!

    2. Wildcat*

      They absolutely have to. This is a model rule, but many states model their legal ethics rules off of them (the state I’m admitted in uses the same language):

      “A lawyer who receives a document or electronically stored information relating to the representation of the lawyer’s client and knows or reasonably should know that the document or electronically stored information was inadvertently sent shall promptly notify the sender.”

      ABA Model Professional Rule 4.4

      1. Loulou*

        Right, but the sender is the job candidate. People are wondering if they’ll also notify the firm that the job candidate did this work for.

          1. Wildcat*

            I want to note that none of these statements are legal advice. OP is only asking about the candidate and I am only writing this to explain to commenters why OP has to take a course of action that might seem harsh.

            1. womanaroundtown*

              As a lawyer, I gasped reading that question. I also thought the question made it clear they were going to notify opposing counsel, but I suppose it might not have been clear to everyone who isn’t also an attorney (or paralegal, or anyone working at a law firm/agency) doing a full body cringe.

              1. Starbuck*

                Yes this part confused me:

                “Normally, if something privileged is mistakenly sent out, we notify the other party that they appear to have given us a document by mistake, and assure them that we have not read it. But I am wondering whether I should contact the applicant first (or in addition)”

                Because it wasn’t clear at first that “the other party” means the opposing counsel and not the sender/candidate. So I wasn’t sure who was meant to be contacted second / in addition to the sender.

              2. Dramatic Intent to Flounce*

                I took paralegal coursework, even if I’m not currently employed due to disability, and yep, full body cringe. MASSIVE ethics violation.

                My read on ‘first or in addition to’ was ‘notify the applicant ahead of time, or simultaneously, or not tell them at all.’

        1. fhqwhgads*

          No I don’t think that’s in question. The letter says they have to tell the firm so they’re going to. The question is whether to tell the applicant directly first (and most likely by “first” we’re talking like…within a couple minutes). In other words: this applicant is going to know they fucked up. Do they find out when the place they applied to tells them or when their current firm comes down on them like a ton of bricks.

    3. L.H. Puttgrass*

      Yes, and this is so obviously bad that I’d approach the candidate as if it were unintentional.

      “Hi, Jane. FYI, it looks like you accidentally sent us a confidential memo as your writing sample. Because it’s work product relating to the ongoing Wakeen Teapots case, we’ll obviously have to notify [opposing counsel]. It will be hard to avoid talking about how we received the memo, so if your firm isn’t aware that you’re job hunting, you might want to prepare for that conversation.”

  15. Akcipitrokulo*

    OP3 – that’s actually performing a useful purpose!

    Steve has asked a group if someone can do something.

    Joe responds to him saying he will.

    By saying “Thank you, Joe”, Steve is letting the group know it’s been taken care of, and no-one else needs to respond.

    1. Green great dragon*

      But by responding ‘I will’, Joe has already informed the group it’s being taken care of!

      Steve’s message doesn’t add anything that I can see.

      1. Banana*

        I frequently interact with a technical group that is full of very smart people, but people who often don’t have the best grasp of our (non-tech based) company’s challenges, goals, and objectives. Part of my role is helping them understand those things and also bringing business problems to them to solve. Usually I am interacting directly with one person, but a broader group of more senior people is copied. Replying-all with a thank you confirms that the person who is helping me does have a sufficient understanding of the issue at hand and their solutions are acceptable. I am very likely to respond instead with a correction or redirection, and the senior people are anticipating that I might, so the thank you is a signal to them as well as an appreciation of the help.

      2. OhNo*

        Not necessarily! The “Thanks, Joe” message confirms to the rest of the group that Joe is, indeed, handling the issue, and presumably would therefore be the point of contact for questions. Otherwise, the message could be, “Thanks, Joe, but Lexi already started on it!” – thereby indicating that Lexi was point-of-contact.

        I can list several situations from my work history, where someone replied-all to claim a task and was assumed by all recipients to be the contact for that issue – only for us to find out later that the volunteer had gotten a private “Thanks, but not needed” message and we actually should have been directing our questions to someone else entirely. As inconvenient as reply-all can be, a message like this can actually be helpful for making sure everyone is on exactly the same page.

    2. RabbitRabbit*

      Joe responded to the whole group saying that he would, not just to Steve. The thank you response from Steve could go just to Joe.

  16. Not Australian*

    “That’s an excellent suggestion, Miss Johnson – perhaps one of the *men* would like to make it?”

  17. Ellis Hubris*

    #4 my experience with employment lawyers is they are worth every dollar. They simply know the law better, likely even that your employer, and have the all important *sense of how it works* – get a lawyer. I’m a paralegal and can figure a lot out myself. Everytime I’ve hired an attorney has not been a regret. My only regrets have been the times I thought I had it handled, or an attorney friend from another field thought they had it handled. Experts can answer your questions easily and quickly and correctly.

    1. Richard Hershberger*

      My church is currently involved in renting out a building. We paid a commercial real estate lawyer to draft the lease. The lawyers in the church insisted on this.

  18. Virginia Plain*

    You know the scene in Airplane when someone slaps a hysterical woman, then the camera pans back to show a queue of people brandishing weapons? I feel like the commentariat version of that will manifest, re the community member in #1. And that is not a criticism! He sounds like a complete arse and I hope for OP’s sake that something or someone makes him stop it.

    1. Artemesia*

      when a meeting leader lets someone ride roughshod like this, the other community members are likely also really annoyed and in this case, women in the room are probably getting increasingly hostile that the meeting chair doesn’t deal with this. The delicate thing is that when a presented chastises an audience member, they risk the audience closing ranks and identifying with the chastised person. In this case — the community may feel attacked if jerko is ‘attacked’ — so it is delicate. But needs to be done preferably by the chair and very matter of factly. ‘Carl, Rachel is our project lead, so let’s let her answer that.’ Don’t let the community member run the meeting from the seats. It makes the presenters look weak and disorganized.

      1. sb51*

        Yep. And if the moderator is female and being ignored, having a game plan as to how keep moderation flowing is important. (Will also help when That Guy who ignores ALL moderators with his “this is more of a comment than a question” gets ahold of the mic.)

  19. Paul Pearson*

    Oh I loathe reply all – but then in our office it’s more like this:

    Fred: Asks for info, send-all
    Joanne, Michael and Susanne: Reply:all “I don’t know” (whyyyy, whyyy would you do this?!)
    Elliot: Reply-all with answer. He is wrong.
    Mark: Reply-all correting Elliot. He is also wrong
    Elliot & Mark: 20 reply all emails arguing over who is right
    Joanne: Reply-all to repeat that she still does not know.
    Susanne: Reply- all to agree with Joanne
    Sarah: Reply-all to correct Elliot and Mark. She is also wrong.
    Joanne, upset by all this arguing: Reply all with a fuzzy kitten picture with a trite feel good message.
    Multiple people: Unnecessary kitten cooing in reply all.
    Brian: Reply all with the ACTUAL true answer
    Fred, Elliot, Mark, Joanne, Sarah: Multiple Reply alls to tell Brian he is wrong. He is banished in shame for his obvious wrongness.
    Dark Lord Charlotte, Master of all She Surveys, Manager of the Lives of Lesser People: Reply: ENOUGH! I WILL EAT THE SOUL OF THE NEXT PERSON TO REPLY ALL

        1. Paul Pearson*

          We do not criticise the Dark Lord Charlotte, Master of all She Surveys – or say her name three times.

      1. Koalafied*

        My report, who gets way less email than me, will occasionally reply to an email sent to both of us within 15-20 minutes just to say, “Hmm, I don’t know this one – Koalafied?” I do my best to chalk it up to her quirk, maybe that’s how she gives herself mental permission to file the email in her own inbox, but when I’m drowning in unread emails it can really stress me out by making it feel like there’s more pressure on me to respond ASAP. It which would be fine after 24 hours, or if the question asked for a same-day response and the end of the day was approaching, but after 15 minutes it’s just another email getting in the way of my being able to deal with the original question.

        And as with the original question here, the time burden isn’t a few seconds for a single email. It’s the cumulative impact when it happens regularly, and when you’re already struggling to manage your email volume so every email does make a difference.

        My company doesn’t have a reply-all thank you culture, but it does drive me crazy when an all-department email goes out to 150 announcing a new hire, whom 100 of the people may never actually work with directly, and then 12 people reply-all to add their own, “Welcome!” Call me a curmudgeon, but we are too big and hire too frequently for every new hire to generate 1 potentially useful message (the actual introduction) and 11 completely irrelevant ones. You can certainly welcome a new team member in a private reply without needing everyone else’s inbox to fill up with your performance.

        1. xd*

          I am guessing that this is just because she doesn’t know if you are expecting her to answer or if you will do it, and she wants to get ahead of the fact that she doesn’t know the answer and is hoping you’ll respond and not waiting for her to handle it. I say that as someone who has been in that situation, and while that’s not usually how I’ve addressed it, I understand that it can be uncomfortable. If you don’t like how she is going about it, maybe it’s worth speaking with her and just noting the pattern and asking why she does it and suggesting a method that does work better for you.

          I’m with you on the welcome emails, though! My company now sends the message to the new person and BCCs the rest of the organization, which helps tremendously.

          1. Loulou*

            Yeah, replying all to say you don’t know can sometimes make sense! I will often avoid answering a question if I think X teammate is in a better position to answer it. If that person says they don’t know, I’ll chime in.

        2. Allonge*

          I have a person like this too. And they chat me after 5 minutes if I am still not responding, asking if I will answer. Which I always do, just, in some time as I am handling something else.

          Ugh. And yes, I also thought this is the way they handle some kind of mini-panic on an email they have they have not responded to.

      2. Paul Pearson*

        I am completely bemused why people do this – reply all to say they don’t know or they can’t. Like “can someone take my evening shift?” why would you reply all to say “I can’t!” I mean, why would you?

        1. Colette*

          Because you don’t realize it was sent to a group, or because you think others on the list expect you to answer.

          1. Loulou*

            Or because the other people on the thread will factor your answer into theirs! If I have flexible plans and no one has replied yet, I’m waiting to see if they’re free. If everyone else replies that they can’t, guess I’m canceling my plans.

            1. Sola Lingua Bona Lingua Mortua Est*

              Or because the other people on the thread will factor your answer into theirs! If I have flexible plans and no one has replied yet, I’m waiting to see if they’re free. If everyone else replies that they can’t, guess I’m canceling my plans.

              If you play Euchre, think of “censor the dealer. I’ve worked places where the last person to object or claim they have a conflict is on the hook anyway, so a request like this would get an immediate flood of randomly worded “not it” responses.

        2. Jackalope*

          I’ve had times when I gave a Reply All on a thread like that if no one had answered after the usual response time, as a way of bringing it back to the forefront of everyone’s minds. We often have urgent questions that need a response within a minute or two, and so if no one responds the original person will continue with some sort of “Am I screaming into the Void here?” response if nothing is heard soon. Even just an I don’t know response lets them know to look somewhere else.

        3. Merrie*

          So that person and everyone else knows that you’re off the list. This kind of thing happens constantly at my work because it could be one of three different managers, or the person who themself has to miss work, looking for coverage. The “I can’t” reply all means that if Barb has tried to get her shift covered and not been successful, she will then later come to me and say “Well, I tried to get coverage, and Thelma replied to say she couldn’t, but nobody else has, so maybe see if you can bug Milo or Persephone about it”. If Thelma had just read the initial question, thought to herself “Nope, can’t do”, and not responded, or responded only to Barb but later Lionel found out and started trying to get the shift covered, she might get another inquiry later.

      3. Critical Rolls*

        Yesssss. The reply-all “I don’t know” instantly ages me. I slouch over my keyboard, in need of a cup of tea and a lie-down.

      4. Artemesia*

        Reminds me of the Amazon answers to queries where person after person says ‘I don’t know, I haven’t used the X yet’ or ‘it was a gift so I don’t know’ etc etc. — can no one see a question and think ‘I don’t know, so I won’t answer.’

        1. NotMy(Fancy)RealName*

          That happens because Amazon emailed them the question and they don’t realize that it’s for the website.

        2. pancakes*

          I think a lot of the people who post stuff like that are responding to a prompt in an automated email and have no idea they’re posting it publicly.

        3. JSPA*

          I have a friend who legit believes there’s a person asking for her help, and it’s not merely rude but unkind to ignore them. Sigh. (And yes, she’s also scam-bait.)

    1. Happily Retired*

      Every now and again, I am reminded why I chose this user name.

      Thank you, thank you, thank you.

    2. Stay at home dog mom*

      How I love the e-mail that goes to the wrong, and of course large, distribution group, then the ensuing…. “take me off this list” “why did i get this” “quit replying all” 100s later… “for the love of all that is holy, quit replying all”

      so glad outlook has the ignore conversation option!

  20. Lost academic*

    I don’t love a reply all thanks but there’s a critical value in that it refrains from introducing an email to the chain without every recipient. If you tend to reply to the most recent email, you can start inadvertently excluding people. Just because at this time maybe the matter is closed doesn’t mean it won’t reopen for some reason and when side conversations appear in email threads it’s a problem at times .

  21. Karia*

    The year thing was probably entirely deliberate, unethical, awful, and designed to save the company money. I’ve seen a thing in my industry where companies will mysteriously layoff or fire people at 23 months, because at two years employment protections kick in.

    1. blackcat lady*

      I agree. The company knew damn well what it was doing – attempting to save money. I hope you find another job with a firm that actually has a moral compass. If you are in the US follow up Alison’s suggestion about the WARN act. Do you know of other people in the same situation? I’d call employment lawyers and check their fees.

    2. ecnaseener*

      It could only have been deliberate if the company hired a bunch of people at the same time as LW. (Which I know does happen, but I’d think they’d have mentioned in the letter if their whole cohort was affected.)

    3. Snow Globe*

      Likely illegal violation of the WARN act, but if 100 people were laid off at the same time, I don’t think it was a deliberate attempt to lay off people within a month of being vested in their 401(k).

      1. MCMonkeyBean*

        Yes, it sounds like this is just extra unfortunate timing for OP in an already unfortunate scenario. It sounds like her direct bosses are trying to do what they can to help but haven’t found anything yet. If they are violating the WARN act then hopefully OP can get an extra month of severance out of that (if I understood correctly that they are currently getting 30 days but are required to get at least 60 if the layoff was sudden and immediate). That definitely seems like a significant reason to seek a lawyer.

      2. Here we go again*

        A former employer of mine is getting sued because of warn act violations during a bankruptcy. They declared bankruptcy on March 5th 2020 and everyone was promised employment and full benefits through at least may 5, 2020. Guess what excuse they used three weeks later to fire everyone and terminate benefits immediately through email at 10 pm? Yeah I hope the lawyers really screw everyone at that private equity firm who called the shots on that.

    4. A Penguin!*

      Unless OP5 was part of a sizeable cohort all hired together I don’t think this timing was an intentional money-save. With over 100 (former) employees affected, it’s probably close to impossible to hit a timing that isn’t short distance ahead of somebody’s anniversary. Assuming random distribution of anniversary dates there’s likely 1-2 more who missed the cutoff by a week (there should be an average of one anniversary every 3-4 days).

      I’m not disputing that money-saving layoffs at close timing don’t happen, just that absent details that aren’t in this letter I don’t think it’s the case here.

      1. doreen*

        With 100 layoffs, there probably were at least one or two more who were laid off within a week of an anniversary – but that still leaves a good chance that the LW was the only one laid off within a week of the first anniversary. Someone laid off a week before their second or third or fifth anniversary would lose a week’s pay – only those laid off a week before their first anniversary would lose the company 401K contributions and the LW may have been the only person in that situation. It is not unheard of for employers to make decisions based on timing and those decisions may not be legal * – but it’s unlikely that one would decide the date of a 100 person layoff based on one person’s vesting date.

        * My father was laid off from a job in the late ’70s and the Federal judge in the lawsuit ruled that the company deliberately laid off the people who were about to qualify for a particular pension benefit. But it’s a lot easier to come to that conclusion when it’s a few thousand people being laid off , all of them shortly before they would become eligible for a benefit and there was documentation that some plants were shut down simply because a lot of workers at that location would soon qualify.

    5. Dragon_Dreamer*

      I was let go from a job at 88 days because the manager thought that would prevent unemployment from being paid out.

      He’d just taken over the motel management, and within 2 weeks fired all the women except the one he was sleeping with. (He was friends with the owners and very sexist. He actually sided with an ex-friend of mine, also employed there, when the ex-friend screamed at me for not wanting to sleep with him. Big incel.)

      I was fired after a weekend where I’d been ordered to create a poster for an event, off the clock. (Which I did do.) The manager then told me on the phone not to bother coming in, as soon as I’d emailed him the poster.

      He tried lying to unemployment about why I was let go, claiming a “bad attitude” and making nasty remarks about “lazy women.” He had no documentation to back him up, so the judge told him to go pound sand and awarded it to me.

      The other women sued for discrimination and won, but my unemployment ended up being more than they eventually got. Just because you win doesn’t mean you’ll easily collect. :/ He ended up losing the owners their franchising rights, and the motel had to be sold to another brand.

      Quick addendum: While I was still employed there, the previous good manager had asked me to brainstorm good ideas for partnering with local businesses in my small town. I’m related to half the shopkeepers and went to school with the kids of the other half. I’d made a list, things like chocolate samples from the local chocolatiers on the pillows, a list of places to order flowers, special coupons, etc. The important thing was that the motel would PAY for these samples. I’d also made up information packets on local attractions.

      So imagine my surprise when I got a phone call from several local shop keepers, telling me that this new manager had called them, demanding full vases of flowers, tickets, and large boxes of chocolate FOR FREE, saying that I had promised him they’d donate it all!

      Thankfully, they believed me over him, but I was still extra nice to them for weeks after!

    6. Miss Pantalones En Fuego*

      Absolutely this, I suspect. I used to work for a place that let every single new employee go the day before their probation period was up. I only worked there for a few months myself, as of course it happened to me too, but in that time everyone who had been there before me got let go. I can only speculate that it was to avoid having to pay benefits and so on but I wonder how much they actually saved.

      1. NoviceManagerGuy*

        What was the job? Onboarding people is expensive and they’re not very useful for the first week at least. Bizarre.

    7. Artemesia*

      I know two people who were hired in start ups with stock as their major compensation and fired literally the week before it would vest — and after they had done major important work to get the company up. In one case, it was a startup of an on-line business and the person had designed and built their entire web presence and then was let go after a year a minimum wage, the week before his stock share kicked in. Firing people to avoid compensation that is promised is not ‘good business’ — it is appalling.

    8. Irish girl*

      There was a company around me that instead of massive layoffs, they found the smallest reason to terminate people and then fight their unemployment. People who worked 20 years with no issues all of a sudden written up 3 times in a month. My MIL was one of them. Went from high performer with great review to fired for performance less than 6 months later. She ended up winning her unemployment and meet at least 3 other people in the same boat waiting for her case to be heard. Oh and they did hire a bunch of people that were younger and paid them less.

  22. Karia*

    LW1: I hope you can resolve this. I also wonder if the reason there are no female community members there is because of men like Floyd.

    I’ve been in this situation at a local meeting where men were allowed to go on rambling monologues (one was literally thirty minutes) but if women tried to talk more than once or twice they were admonished to ‘let other people have a say’. As a community member I decided to stop wasting my time where I clearly wasn’t valued, and never went back.

    1. ecnaseener*

      But the letter doesn’t say there aren’t any female community members, only that the govt employees are all male apart from LW.

      1. Starbuck*

        The general point stands that allowing this sort of behavior to continue totally unchecked is probably alienating at least some of the women who come to these meetings, if indeed there are any. I’ve seen/experienced as much myself. If I’m attending and see that even the official government women can’t get any respect, I’m going to assume no one’s going to intercede on my behalf either.

  23. QuickerBooks*

    I noticed that no one is questioning Steve’s original email. It’s possible that OP is bothered by the reply-all “thank you” because the original email never should have been a group email to start with, and instead should have been an individual email to the one or two people most likely to have the answer.

    Yes, there are a million zillion cases when a large group email is appropriate. There are also a million zillion cases when it’s not. I know that I am more likely to be annoyed by little “thank you” type responses in cases where I never should have been copied in the first place.

    1. boo bot*

      Yeah, I feel like this is absolutely an issue of the medium and not the message; a smaller group might need to be targeted, as you say, or I was thinking that this kind of request might be better suited to something like Slack or a similar messaging system, where every reply doesn’t pop up in people’s inboxes, but the whole conversation is visible to anyone who needs to see it.

      If people are constantly using a system in a way that’s unproductive and annoying, it’s worth asking whether there might be a better system.

    2. Xantar*

      That’s what I was thinking. The “thank you” email at the end is a signal that the question has been resolved and OP doesn’t have to spend time answering it. But if OP was never going to respond anyway because they don’t have the knowledge or it’s not their department or whatever, then the real annoyance is with the original email.

      I also have this issue when my workplace sends out, “It’s Wanda’s 5 year anniversary!” followed by umpteen “Congratulations Wanda!” emails, but that’s because I don’t know Wanda (my workplace has hundreds of employees).

  24. SleepyKitten*

    #3, You can set outlook or gmail so that the default option is “reply all”. I do this because accidentally leaving someone in and briefly annoying them is better than accidentally leaving someone out and having them miss crucial information. I’ll bet Steve has this setup too.

    1. Teacher, Here*

      I just full body shuddered at how awful things would be around here if reply all was the default. And also how often people would see things that were… not intended for them.

  25. Popinki*

    Ah, local government, the land of jerks who think they can do anything they want because “we’re paying your salary!!!” and higher-ups who won’t do anything about it for fear of losing community goodwill. I have no advice, but much sympathy.

    I regularly get people threatening not to vote for me in the next election, even though my position is not and has never been elected, and it’s a job literally no one else wants. I was the only applicant for it…

    1. kittymommy*

      I feel every single bit of this as a fellow local government employee. And depending on organizational structure (elected officials or not, how high up) pushing back on the misogyny from a citizen is not really an option. Luckily, the elected officials I work for are aware enough to notice this and would probably(and deliberately) intervene by sending the questions back to the LW, but that’s not a given everywhere.

    2. Brett*

      My favorite was always the people who called their local state legislature rep to call me up to tell me I was going to be fired. “I know people and I’ll make sure you don’t have a job by Monday.” Yeah, except you are in the wrong government and the people you know (i.e. other elected officials from your party) can’t just up and fire a merit employee that they have no role in appointing.

  26. Batgirl*

    OP1, a lot will depend on the reactions of your colleagues in the moment – are they pausing, or looking at you before answering the community member? If so that’s a really easy opportunity to say “Actually, I will just finish what I was saying before I pass over to Paul”. If not, I would start pointing out overtly, that you are being interrupted. Something like “Well, I will just finish what I was saying before taggging in Paul” or “I was actually interrupted halfway through a sentence, so I will finish speaking before we get Paul’s take as well.”

    1. All the words*

      And chat with Paul ahead of time so he’s aware of the problem and his answer can be “what Rachel just said”.

  27. Workerbee*

    Op 1, just because this lout interrupts you or heads you off at the pass to call in Male Colleague instead, it doesn’t mean you have to listen to the lout, acknowledge him, or (in effect) obey him. After all, how often do men like that shut up? And he’s already not listening to you, so you can dispense with any perceived courtesy in the world and just keep talking. No apologies necessary to him, either. If you do get hung up on courtesy, tell yourself you are doing him the courtesy of enlightening his life and it is on him if he chooses to be wrong.

    This takes effort, training in the privacy of your home, willingness to let the sweat pour and the breath heighten until you get used to it, but—if you are the person who should be answering that question, letting that jackhole undermine you is just not acceptable. Plus risks having your other colleagues start to think of you in the same way. Folks unfortunately have a way of latching onto negative perceptions easily, particularly in groups.

  28. Elaine Benes*

    LW1: Could you start responding back in the moment with an interrupting-isn’t-the-way-we-do-things-here vibe? ie: “Hey Bob, I appreciate that you’d like to hear from Paul as well but you need to ask for that at the end of my answer instead of interrupting me. Please don’t interrupt government members, and hold your questions until the end.”

    TBH, I’ve never been to a town forum or any other local government meeting that doesn’t have *very* clear rules around who can speak when. I’m sort of surprised your group doesn’t already have that established (might be worth bringing up with your manager?).

    1. Brett*

      The lack of rules is because it is an informal forum for feedback. This is basically where people who are doing the technical work and project management sit down with key stakeholders and influencers from the affected communities and try to get their support behind the project as proposed.

      Having been on the government end of dozens of these meetings in the past, quite honestly the real purpose is to shut off the possibility of lawsuits and media ambushes in the future on a project. A stance of “Please don’t interrupt” is going to be counter-productive (it’s not a formal testimony/presentation situation). What you really want is for the community members to air their grievances and address them up front. And collect information so that when they go to the media or bring in the lawyers, you are already prepared for them and don’t end up derailing the project. Or can make changes early to avoid legal/media problems later.

      1. JSPA*

        Yup. It all ends up being an exercise in CYA.

        The government folks are mostly there to record it all while maintaining reasonable composure in the face of:

        random venting; legitimate grievance; fan-fiction level what-if scenarios; outright word-salad; the airing of tenuously related or entirely unrelated longstanding inter-group grievances; commentary-as-social-outlet (or psychological need); grandstanding by would-be budding politicians; public participation as a philosophical ideal; flattery from people who are looking for friends in high places to provide cover for questionable activities; (often well-informed but tonally-problematic) sniping by people whose preferred option / scenario / use / contractor has already been passed over or ruled out; students who are getting credit in a gov class for “participating in government and documenting it”; people who mistook the listening session for a council meeting, and spend their time complaining about who isn’t there (city council members, despite this being a county issue, or vice versa; local magistrates, who are magically supposed to rule on whether this is “all legal”; the mayor, the county executive, the president of the US, the previous president of the US, the head of IBEW local whatever, the local big name oligarch whose company does anything tangentially related or physically adjacent, as well as all news organizations past, present and future).

  29. Elsa*

    LW 3 –The one that gets me is when someone send an email saying “I’d like to compliment all of you on your hard work to make the Llama Grooming Festival a success. You’re all geniuses!”
    And then comes a barrage of reply-all “thank you”s.

    1. I should really pick a name*

      Corollary: Every man needs to learn to let her finish so she doesn’t have to say it

      1. Dr. Rebecca*

        ^^^This. Particularly if we’ve become so frustrated that it leaks into our tone, because there are days when my “let me finish” will land with all the force of “shut the **** up.”

    2. Curiouser and Curiouser*

      And then she’ll be spoken to by her manager about her “tone”, no matter how pleasant she sounded.

      1. ScruffyInternHerder*

        I figure if I’m going to get a talking to either way, then “let me finish” and “I was speaking” are going to land with “shut the actual eff up” force.

        Thankfully my boss does not police my tone.

  30. NotAnotherManager!*

    #4, that’s a huge mistake and not one junior attorneys should make. When I worked in legal, we trained every single person in the building, attorney or not, on client confidentially and that violating it was a terminable offense on the first instance. This wasn’t even an inadvertent disclosure, she submitted it as a writing sample!

    Ethically, you’re obligated to notify opposing counsel and confirm that you’ve purged every copy from your system. It would be nice to give the applicant a heads-up, but I expect they’re not going to be hired by you or their prior firm and will have trouble with references in the wake of this.

    1. Fashionable Pumpkin*

      Do you think it’s possible that they thought this info might be… welcome by the opposing team? I would wonder if they hoped it would give them a leg up over other candidates, in a “I can bring this inside info to you of hired…” kind of way. Super unethical, but I’ve noticed unethical people tend to assume everyone else would be just as unethical as them, if given the opportunity.

      1. NotAnotherManager!*

        I’m more inclined think it was careless/lack of experience rather than malicious. Client privilege is such a big deal in law that you’d really need to be sure the recipient was open to partake of your unethical behavior, and, really, if you’re going to go the “inside information” track, you do that verbally (and directly to an attorney on the case) and not with a paper trail through recruiting/HR. So, even if the receiving firm was unethical, they’d not want someone so stupid about how to be unethical in their ranks.

        1. cynara c*

          But confidentiality and such is very basic stuff, it’s fundamental to the work of most if not all attorneys (I’m trying to allow for instances where an attorney may not be representing clients). And doing conflict checks is basic stuff. This is the kind of thing that in my *paralegal program at a community college* was drilled into us in all of the fundamental courses. So if it was a careless error…I really do have to ask if this candidate is ready to practice law. Eeeeek.

          1. NotAnotherManager!*

            Agreed. I wouldn’t hire this candidate and would likely put them on a permanent do-not-hire list based on the stunning display of poor judgement. I used to manage paralegals, and client confidentiality and what, practically, that means was part of onboarding training.

            I just generally find that incompetence is a much more common cause than malice in most situations. And, again, if this was some ham-fisted attempt at slipping info to the other side, that’s bad on even more levels AND poorly-executed. This woman has shot her legal career in the foot before it even started.

      2. Wildcat*

        Op’s firm would be in so, so much trouble if they did this. Had they hired this person, she would not have been allowed to do any work whatsoever on this case.

        1. Turanga Leela*

          If the firm had hired her, it might have had to withdraw from the representation. I’ve seen that happen.

    2. Ursula*

      What is the firm expecting her to submit as a writing sample? I would assume most of what a lawyer writes is privileged.

      1. NotAnotherManager!*

        If you redact the identifying information and have permission of the firm that owns the work product, you could use something like what she submitted. You can also submit something you wrote for a law school class, moot court, law review, or even a public pleading you drafted from a law clinic your law school sponsors. Hell, make up a fake dispute and write something on that.

        This candidate failed on three fronts – first, leaving identifying information in the document (this can be names, places, or enough of the facts to make the case identifiable to those in the know); second, not running a redacted version of the memo by the appropriate party at the firm for which it was written to confirm that it’d been anonymized enough to be submitted; and, third, by submitting it to the firm for which opposing counsel works.

      2. Wildcat*

        I personally used a judicial memo I wrote as an intern. Even though everything was publicly filed, I changed all names.

      3. Turanga Leela*

        You can submit an article or a brief that’s been publicly filed, although students usually don’t have briefs that they’ve written on their own. It’s common to use a law school assignment (as NotAnotherManager! says) or a memo that your supervisor has approved of your using.

        I used to occasionally use a memo that I probably shouldn’t have, because my employer hadn’t signed off on it. It was a summary of the current policy in an area we worked in—just my findings, no recommendations, no relationship to any client or litigation, no confidential info about my employer. It was the kind of thing I’d have shared with other organizations in a heartbeat, which is probably why I felt fine sharing it as a writing sample. But in retrospect, it was a questionable choice.

      4. Elysian*

        When I was a young attorney applying around, I used a publicly filed version of a brief that I was the primary drafter on. Even though it was publicly filed, I still redacted the names of the parties (someone who cared could have looked it up, I suppose, cause it was public and file-stamped). I had a cover letter that noted the brief had been reviewed by senior attorneys at my firm and lightly edited prior to filing. I did not clear using it with my firm because I was apply to leave them and I didn’t want them to know. I spent a really long time debating how to do this, because otherwise I would have had to use a writing sample from law school and they just weren’t as good or indicative of how my writing had evolved. I felt lucky that I litigated and that my firm was small enough that my name was on publicly filed documents; if I had a transactional practice (drafting contracts, etc) I’m not sure what the heck I would have used in this situation!

  31. Mannheim Steamroller*

    OP #1…

    Maybe the solution is to attend the next meeting without your staff or your manager, so that you will be THE representative from your agency and Floyd will have to let you answer.

    1. lost academic*

      Or have them go without her and every time that person asks a question respond with “OP needs to answer that, next?”. Literally every question from him.

  32. anonymous73*

    #1 I slightly disagree here. Yes I think OP should talk to her boss/team* abut having her back, but I also think she needs to say something in the moment instead of having her male team members stand up in her defense immediately. A simple “Please don’t interrupt me” should be enough and if this jackass still makes a comment about letting someone else answer, THEN one of your team members needs to back you up. I know you don’t want to make waves, but I feel like someone who won’t let a woman speak to something she knows about will just have more negative commentary if one of her male colleagues defends her initially.

    *if your boss/team don’t see anything wrong with this or won’t have your back, that’s a bigger issue.

  33. Esmeralda*

    OP 3. Alison, it may not bother you to take a moment to just delete those messages, and I wouldn’t mind either, except it’s not a moment.

    Steve sends request. Tia responds-all, Sorry I can’t cover that shift, I’m out of the office. Steve responds, That’s ok, Tia! Thanks for letting me know! Then Jens replies, I think I can cover for you! Let me check. Steve replies. Sue replies that she can cover if Jens can’t. Steve replies. Others who haven’t read the string reply. Steve replies to each of them. Jens comes back. Steve replies. Somewhere in there someone also says, Hey everybody, the deadline for the TPS reports moved up. Everyone replies. If I don’t read the string, I don’t know about the TPS reports. So I have to read the freakin thing every time someone replies. Even if I wait til lunchtime and read the whole string, I’m reading the whole string. Aaaarrrrrgggghhhhh.

    And if anyone decides to change the subject header when they respond, I now have two strings…

    1. jane's nemesis*

      What you’re talking about is extremely annoying, but is not the same thing as what the LW was writing about, which is a one-off “thank you!” not a huge reply-all day-eating thread.

      Though I totally agree with you about the AAAARGGGGHH of it all.

      1. Unaccountably*

        It’s equally annoying when fifteen people on the thread all feel that they have to send their own public “Thank you!” because of *insert posts above about why thank-you messages should go to the whole entire email chain*.

  34. anonymous73*

    #5 One of the 3 times I’ve been laid off, my company actually went the extra mile for me. Severance was based on the number of years I had worked there, but I had started as a contractor and only worked as an FTE for 3.5 of 5 years. I only expected to get severance based on my FTE time, but they based it on 5 years. Your company SHOULD have based your severance on 1 year since you were so close, but I’m not sure that they HAD to do that. I would consult a lawyer just to make sure you aren’t missing something, but this may just be one of those things that has no satisfying solution.

  35. generic_username*

    #3 (reply all thank yous) – I’m guilty of doing that. It’s actually been a concerted effort to do better or to include “loop closing” sentences in my thank you as well

    #5 – definitely ask for more time to consult a lawyer. Maybe they’ll rethink their decision not to give you the extra week – that’s such a crappy thing to do

  36. pay people for their work*

    LW #2 I guarantee your firm is billing their clients at attorney rates for whatever work you are doing that might otherwise be done by a lawyer, and you are not seeing the benefit of that additional fee.

    1. lizesq*

      I truly doubt this. Most paralegals use the same billing software as attorneys and bills are generated by the rate of the person who enters the time. And even if this was true, what difference does it make at all? LW’s job is to support the attorneys they work for, if they have an issue with compensation for the work they’re assigned, they have a fundamental issue with the job they have.

      1. PayGradeOP*

        It’s not ethical for an attorney (or any legal professional) to bill for anything more than hours they work. There are certainly unethical attorneys out there, but my firm is very reputable and that is why I continue to work there! I bill for the hours I work; the attorneys bill for the hours they work. So, yes, the work I do is at a lower rate than an attorney, but my question was truly about how I could graciously remind them when I was asked to do something outside of my job title (i.e. an attorney task) without being too abrasive.

        I am paid very fairly for my role/title – it’s just I often get asked to do things that are way beyond my level.

        Co-workers would say “that’s beyond my paygrade” to indicate that’s outside of their role/position, but it didn’t mean they didn’t get paid enough in their position. Just that they weren’t lawyers so they weren’t going to do the lawyer stuff (even if they did work at a high level for their current position).

    2. Lalchi11*

      No. Unless her firm is hugely unethical, when clients engage a law firm to represent them, the separate rates for partners, associates and paralegals are all agreed to. When paralegals work on the case, they input their time into the billing system and then it applies the agreed paralegal rate to the hours billed.

  37. suggestion box*

    LW#3: our group made an email norm that if you feel the need to send back a thank you only, you put it in the subject line so people know that’s all there is and they can delete it or simply mark at read.

    So for example:

    Email #1 subject line: Question about Project X
    Email reply subject line: Re: Question about Project X
    Final email reply subject line: Thank you Joe! Re: Question about Project X

    It has saved us a lot of mental aggravation.

  38. Tacotsunami*

    Re: Number 3. If I reply all with a thank you, it’s usually strategic and Alison nailed the reason there. If I’ve requested information from a group and one person has replied with what I need, I’ll send my thanks in the same format to close the loop for the larger group and reply more in depth to the correct person off thread.

    Likewise when I receive those thank you’s, I view it as “case closed”, clear the brain space and move on.

  39. Three Goblins in a Trench Coat*

    LW #3, I’m definitely in that camp of replying “thanks, Joe” to signal to everyone else in the thread that I’ve got the answer I need and they are off the hook. Granted I work on the client care side of the software world so it’s not always obvious to the people in the thread that’s the case. If it were more cut and dry, such as “how many llama’s are in the pen” and the answer is a clear 42, I probably wouldn’t feel the need to reply-all and let everyone know I’ve got the info I need.

  40. Purple Cat*

    LW2
    An alternative to “It’s above my pay grade” is “Sorry, that’s outside the scope of my role. Check with Laura the Lawyer instead”.

    1. SweetestCin*

      That’s a really good script for things that aren’t so much above your pay grade, but are completely outside of your role. Due to some partnerships, I get a lot of requests to do A, B, or C, because that’s what people at X company do with my job title. Except at company Y, where I work, we don’t do A, B, or C and I don’t have the permissions or programs or know-how to do it.

  41. Big Bird*

    Assuming you are in the U.S., the Department of Labor would be interested in this case as a potential example of someone being deliberately terminated just before vesting. My employer carefully checks with its benefits office before terminating people (especially in a lay-off) just exactly to make sure this does not happen. Given the numbers involved and the carelessness of the action, it might be that you are not the only person affected.

    It would also behoove you to request a copy of the 401(k) Plan Document and the Summary Plan Description, ASAP. There may be a loophole.

    One caveat–your benefits office can only react to the information it receives on official employment records. So if your termination date is unchanged, then you are NOT vested according to the provisions of your 401(k) plan document. Plans can get into big trouble for making exceptions to the rules if the paperwork does not support a revised termination date. But if the DOL is not successful in encouraging your ex-job to change your termination date and vest you, another potential remedy would be to pay you OUTSIDE the Plan. (And increase the payment to compensate you for the fact that you can’t roll it over into an IRA or to another plan.) A good employment lawyer can make that happen
    https://www.dol.gov/agencies/ebsa/about-ebsa/about-us/regional-offices

    1. Essess*

      I definitely agree about seeing an employment lawyer.

      However, I don’t see how this could fall under “being deliberately terminated just before vesting” since the letter states that 100 positions were eliminated, not just the OP’s. Unless all of the terminated people were approaching their 1-year anniversary, this would be a case of bad timing for one person in a batch layoff and not a targeted layoff. But I do hope the OP pursues checking if they have other recourse, such as Alison’s WARN act and other suggestions.

  42. StrongFeelingsAboutReplyingToAll*

    Re #3: I am of the camp that reply-to-all thank you messages are disruptive and unnecessary. I understand how come of the comments here are viewing it – and maybe this is some context.

    EXAMPLE 1:
    Employee 1 emails multiple coworkers: Does anyone know how I can do X?
    Employee 2: X can be done by ___________.
    Employee 1: Thank you, Employee 2, that is what I needed.

    In this example, I think it is okay and appropriate – it is letting everyone know the Employee needs no further input. There is benefit to the reply-to-all thank you because it truly closes a loop.

    EXAMPLE 2:
    Employee 1: *Emails Employee 2 with a cc to several other employees* Hi Employee 2, here is the file you requested.
    Employee 2: Thank you!

    In this example, replying thank you to all is truly NOT necessary – and probably super disruptive/annoying to everyone cc’d on that email. (This applies to things outside of an organization, too. I often get clients who send me files and cc others at my organization since we are a team – but the cc’d people don’t need to know I got the file and said thank you.)

    In short, I think it can depend on context – but if there’s no benefit to sending the thank you to everyone on the message, please think twice before hitting reply-to-all when all you are saying is thank you.

    (And no matter what – please, please, PLEASE do not reply-to-all to say “You’re Welcome!” There is truly NO situation that is okay!!)

  43. Sola Lingua Bona Lingua Mortua Est*

    In the past, I’ve had colleagues who would say “that’s beyond my pay grade,” but it always sounded really abrasive to me.

    That’s a very different usage for the phrase than what I’m used to. I’m used to that phrase meaning “I don’t get paid enough to own this decision or (in)action.” (e.g. “Can you waive the late fees?” “Do you think they’ll fire me if I call off on Friday?” “Should we reboot the main database server before we leave for lunch and see if that fixes the issue?”)

    1. PayGradeOP*

      That’s part of the reason it *does* bother me. I generally hear it from people who mean that’s way beyond the role I’m hired for currently – but it *does* sound a little bit like it’s a complaint about salary. (The thing is, you could triple my salary but it still doesnt make me a lawyer equipped to perform some tasks!)

      1. pancakes*

        I think you’re taking it too literally, but there are plenty of other ways to communicate “I’m not the right person to complete this” besides that one phrase.

    2. doreen*

      In my experience, it’s used as a little bit of both – it does sort of imply “I don’t get paid enough to make that decision” but because in my work environment “pay grade” was a good proxy for authority it also meant ” I don’t have the authority to make that decision”

  44. Brett*

    LW #1
    Although this could easily be driven by sexism, I think it is also important to consider the possibility that this is personal. John sounds like a typical CAVE person (Citizen Against Virtually Everything), and often they use tactics like this because they have characterized people on the government side as more or less flexible or more or less aligned with their views (which is opposing everything, especially if it spends money on someone other than them).

    The reasons that John thinks Rachel has something personally against him or his views could be driven by sexism, but it is likely he is doing this because he doesn’t like Rachel personally and what Rachel represents. For PMs, this can often be because the PM is a decision maker on timeline or go/no-go, so he wants to use the technical reports (especially engineers) to try to weaken Rachel’s stance before Rachel can establish a firm answer.

    CAVE people are also very touchy to shut down too. Quite often if they don’t get their say in community meetings and don’t get their way, their next step is to file a lawsuit or go to the media. Government projects I was associated with landed in front of the state supreme court over a half dozen times in a decade because of this exact scenario, and even more landed on the evening news.

    1. Mannheim Steamroller*

      I haven’t seen the “CAVE” acronym before, but I’ve dealt with “BANANA” (Build Absolutely Nothing Anywhere Near Anybody), “LULU” (Locally Undesirable Land Use), and the ever-popular “NIMBY” (Not In My Back Yard) through my career.

      In each case, though, they have tended to let women speak (on either side of the podium) without interruption.

      1. Brett*

        The difference is where NIMBYs et al want things kept away from their homes, CAVEs are mostly anti-tax/anti-regulation (they are not even small government, because their target is mostly local government). That’s where they can be different from NIMBYs because they generally do want to disrupt the government side of the podium and tend to have this firm belief that managers, supervisors, department heads, etc are their enemies while the rank-and-file clerks, engineers, etc are on their side. Papering, using massive amounts of sunshine law requests to try to force a local government into bureaucratic inaction, is an example of a common CAVE tactic.

  45. Tee 3*

    Regardless of what is motivating the manager to do what he does (I didn’t automatically assume sexism; I assumed history with the others, tribal knowledge about the others’ potential contributors, etc), the OP doesn’t need to assume a reason in order to find ways to better position herself. Outside larger meetings, have follow-up conversations with the community members individually and impress them with the way she conducts those one-on-ones. Then in larger meetings, reference those one-on-ones and develop a reputation as the most in-the-know person in the room. Win friends and you’ll win credibility.

    1. Wisteria*

      If you are referring to LW#1, the person asking to let a man speak is not a manager, it’s a community member.

  46. Critical Rolls*

    LW #1: Concur that “Please stop interrupting” is a very useful phrase, as it highlights the problem behavior.

    “Actually, citizen, as project lead I’m best situated to answer that. If I can finish? As I was saying…” With no pauses for input.

  47. Observer*

    #3- Thank you emails

    You are spending more energy on this issue than it warrants. Really, how long does it take to get rid of them? If you want to cut down on the time, take a minute and set your email client to show one or two lines of the email without even needing to open the email.

    I suspect that you spent more time writing this letter than you’ve cumulatively spent in the last month getting rid of the excess thank yous.

    Now, if the REAL issue is not the thank you emails but performative thanks within a culture that doesn’t actually allow for people being appreciated, that’s a genuine problem. But then, you are better off focusing on that problem and figuring out whether you can change the culture, learn to live with it, or start strategizing on how to get out.

  48. Verthandi*

    #1 This guy sounds like he’s trying to be a moderator and is failing at it. I’ve been on panels with people who hog the floor, including moderators. I also had the frustrating experience in my early 20s in which I had a difficult time getting words in edgewise, and I thought for far too long that it was me being a terrible communicator. I read stacks of communication books to try to figure out what to do. They all had advice that required getting the floor, never *how* to get the floor. My assumption was wrong. It was only two indidviduals, but most of my interactions were with them and it was that they didn’t respect me.

    There’s good advice above. I’d like to add the following to your toolkit.

    Channel your inner Vice President. “Mike, I’m speaking.”

    You: (beginning to answer a question)
    Mike: (interrupting) I was hoping Paul would answer that.”
    You: Paul, before you answer, I’ve got just a quick thing to say. (say the thing) Thanks, Paul, back to you.

    Good luck!

  49. Delta Delta*

    #1 reminds me of the community forums from Parks and Recreation, and specifically the guy who hogged the floor and said “I have some things to say about Laura Linney.”

  50. rage criers unite*

    #1 – “Well, actually Dick, I am the project manager, so I can answer that one” – continue to answer.

  51. Lizianna*

    #1, I’m a woman in a male-dominated field, and have this happen frequently.

    Unfortunately, it’s possible the men on your team don’t realize this is happening, but I’ve found that once I raise it, they become more aware and usually are willing to support me by redirecting things back to me.

    Second, I would look at how you’re structuring your meetings. Do you have a facilitator who is not also a subject matter expert? It can be really hard to manage interrupters and cover the subject matter.

    I can think of one particular community member who constantly interrupts me to ask questions of my (male, older) staff members. They have gotten really good at saying, “Well, since Lizianna is the lead, this would be her call.”

    Or you can say something like, “Mike, please let me finish, and if you have follow up questions after I’m done, we’ll have time for those.”

  52. Lizianna*

    #3 – I’ve found I got a lot more productive when I stopped letting constant emails interrupt my workflow. I actually turned off desktop alerts, and check my emails once every hour or two. So if I get an email, I don’t have to drop everything to realize that it’s just a “thank you!” or something else non-urgent. If I take emails in batches, it really only takes a few seconds to archive the ones that don’t need a lot of attention, and makes those types of emails a lot less annoying.

    (FWIW, your office culture does matter here, but I haven’t worked somewhere where instant answers are expected to emails. If something is truly urgent, it needs to be an IM, text, or phone call).

  53. Optimistic Prime*

    #3 At my old job we were expected to respond to every email. We were open 7 days a week and when I would come in from my weekend, I would find a barrage of useless replies to a “heads up” email. Then I would have to add my “will do” or something similar to the chain or I would get in trouble. It was extremely grating. So no matter how annoying it is to see a reply to an asked/answered email… it could always get worse. But I hope it doesn’t.

  54. Calliope*

    I feel the need to stand up for hitting reply all to say “thanks.” I don’t think it’s always or even usually necessary, but if the person who has primary responsibility for the thing that was sent replies to say thanks that implicitly tells everyone else that it has been received and is being handled and they don’t need to stay in top of it.

  55. Ann Nonymous*

    #5 I immediately flashed on Ed Norton’s scene in Fight Club where he, um, arranges a nice severance for himself!

  56. OP #4*

    OP 4 here. Thank you for all of the thoughtful responses and suggestions! I’m sorry I wasn’t more clear — there is no question that we will notify the former employer (and no, I did not read the substance of the document — my clients name appeared in the heading). My question was more about whether it is appropriate to contact an applicant we have not interviewed and will not hire just to let her know that she has made a horrible error in judgment and to give her a heads up that we will be notifying her former employer. Sounds like the consensus is yes. Thank you all!

    1. Regular Human Accountant*

      Bless her heart, but she really does need to know. And I’m sure her former employer will want to do some digging to find out how many other errors in judgment she made while she was there!

    2. Calliope*

      Are you sure she didn’t ask for permission and redact anything truly privileged? It seems possible that it’s an obsolete memo that she was given permission to use, though of course informing the former employer will clear that up.

      1. NotAnotherManager!*

        If OP#4’s client’s name was in the header, it wasn’t cleared. Standard practice dictates that you either redact or change the names of the parties, even for cases that have been resolved with no possibility of appeal. The privilege survives the matter, and the privilege belongs to the client.

        1. NotAnotherManager!*

          Sorry, in this case the privilege belongs to the firm because it’s an internal memo (attorney work product privilege) rather than something prepared for the client (attorney-client privilege) and has less of a duration. Nonetheless, the applicant’s disclosure could, with an unfriendly judge, constitute a waiver of privilege, which is a big problem.

        2. Calliope*

          Not necessarily. I’m a lawyer. It depends on the content of the memo and the nature of the dispute. For instance, I’ve given an intern permission to use a memo as a writing sample about what circuit to file an appeal in even though portions of the case were still pending. The portion related to that appeal had concluded. In that case, I did have him redact the client name anyway but I don’t believe it was a requirement. The fact that we had represented that client in that appeal was public information.

          1. Calliope*

            (I guess the other way to think about it is that it is possible it was the other firm’s screw up for not being vigorous enough about redaction and not the intern’s.)

      2. bratschegirl*

        OP specified that this is a memo concerning an ongoing case where they represent the other side. Highly unlikely that it’s obsolete or that permission was given to share out.

        1. Calliope*

          I would never want an intern to send a memo to opposing counsel full stop. But I have given permission to use memos for ongoing matters – our cases can go on for a decade and the issues can shift drastically during that time. So I could see this intern getting permission, sending it to 100 firms, and it just being bad luck she ended up applying to opposing counsel. Probably not the case but I thought it was at least possible.

    3. NotAnotherManager!*

      I think telling her is a favor – she is clearly missing some knowledge on client confidentiality, and that is such an important thing for a new lawyer to be well-versed in. She needs to not send that to other firms either, though I’m sure her internship firm will make that abundantly clear. This is so egregious, I feel like there’s almost a professional obligation to say something.

  57. I Faught the Law*

    #3 – it’s to close the loop so they don’t get a dozen more replies with the same answer, and so no one else wastes any more time on it. Simple as that.

  58. Regular Human Accountant*

    #3 I often reply-all to emails with “Thank you, Joe” because my boss, who is normally copied on the original email, is a bit of a micro-manager* and if she does not see my reply, will follow up to make sure I saw it. So for me, it is performative and I’m sorry and also yes I’m looking for another job.

    *understatement of the decade

  59. Pantsuit*

    LW4 Talk to your firm’s recruitment team to determine what they’re telling applicants. When I was applying for associate positions, several firms stated they would not accept my 1L memo, law review write on, or my law review note. They asked for my unedited work product from an actual client matter – this was always communicated by the recruiting team, though I didn’t know enough about law firms at the time to know they weren’t lawyers. This is obviously inappropriate, but it happens, especially when non-attorney staff are running the recruiting process. If your firm is pressuring/instructing applicants to produce privileged information then you need to know so you can stop it.

    You’re obligated to inform both opposing counsel (as the representative of the privilege holder) and the applicant (as the sender). The applicant should be made aware ASAP so she can avoid sending the memo to anyone else and claw back the memo if needed. Personally, I would probably also inform the law school’s career services office. This never should have happened and the school needs to educate its students better. Even if your firm made an inappropriate request, the applicant needed to say no.

  60. HannahS*

    OP1, I think it’s very reasonable to talk to the other people in the meetings and let them know what you’ve observed, and then request that they back you up.
    Here’s a possible script:
    I’ve noticed in our meeting that when Bob asks questions that I should answer as project manager, he interrupts me and re-directs his question to my male colleagues. I’d really appreciate it if next time that happens, you could say something like, ‘Actually Bob, Rachel is the project manager and she’s the best person to answer your question.'”
    You can also say it yourself in the meeting–once Bob says his thing to Fred, you can jump in and say, “Actually Bob, as the project manager, I’m the best person to answer your question. The reason for the change to the plans is…”

  61. Xaraja*

    I think the email thing really depends on eBay context we are talking about. I work in a B2B context and generally work with large accounts. So reply all thank yous to a customer are often as much to close the loop and let the other people from our company knowi saw the email as anything else. And to let them know i am taking care of their customer, because I’m in IT, so I’m probably copying the sales team. But if I’m talking to someone internally I’m going to be less formal and probably send that thank you privately.

  62. lockhart*

    I had a toxic, awful boss who required us to reply-all with “Thanks” to every single email about something being posted. So like 50 times a day I’d say “Thanks!” to 50+ people. Eventually, I made a macro, and the requirement continues to this day, although thankfully that person is no longer employed by our company.

Comments are closed.